Teoremas 'obvios' que en realidad son falsos

Es uno de los dichos favoritos de mi profesor de análisis real que "ser obvio no implica que sea verdad".

Ahora, conozco algunos ejemplos de cosas que son obviamente verdaderas y que se puede demostrar que lo son (como el teorema de la curva de Jordan).

Pero, ¿cuáles son algunos teoremas (preferiblemente breves) que, cuando se expresan en términos sencillos, la persona promedio afirmaría que son verdaderos, pero que, en realidad, son falsos (es decir, teoremas contraintuitivamente falsos)?

Los únicos que me vienen a la mente son el problema de Monty Hall y la divergencia de norte = 1 1 norte (contra-intuitivo para mí, al menos, ya que 1 norte 0 ).

Supongo, también, que

límite norte ( 1 + 1 norte ) norte = mi = norte = 0 1 norte !
no es obvio, ya que uno 'espera' que ( 1 + 1 norte ) norte ( 1 + 0 ) norte = 1 .

Solo busco teoremas y no su (des) prueba. Estoy feliz de investigar eso yo mismo.

¡Gracias!

Esta pregunta es difícil de responder, porque una habilidad esencial para un matemático es poder volver a sintonizar rápidamente su intuición para que coincida con la verdad. Entonces, una vez que sabes que un hecho es falso, muy pronto deja de parecer obvio.
@NateEldredge Estoy de acuerdo, pero siempre hay alguien que se niega a aceptar eso, por ejemplo, 0.999... = 1 .
Cada conjunto compacto es medible por Jordan (falso): math.stackexchange.com/questions/485336/…
¡Estoy totalmente en desacuerdo con que el teorema de la curva de Jordan sea obviamente cierto! Estoy de acuerdo en que el teorema de la curva de Jordan para curvas suaves por tramos es obviamente cierto; pero también es bastante fácil de probar, al menos en comparación con la versión topológica. (¿Es realmente obvio que el copo de nieve de Koch no tiene algún camino patológico desde el interior hacia el exterior?)
Tal vez, para cada respuesta, algunos lectores quieran compartir comentarios sobre cómo sintonizaron su intuición para hacer que el tema de la respuesta sea obvio (me refiero a hacer que la falsedad sea obvia). Dichos comentarios serían útiles, así como la respuesta en sí. Por ejemplo, tomemos el problema de Monty Hall donde la primera suposición ingenua resulta ser incorrecta a través de un análisis caso por caso de fuerza bruta. Se puede obtener una intuición afinada a partir de cualquier explicación que no use el análisis de fuerza bruta, por ejemplo, se podría usar la regla de Bayes
“Toda afirmación verdadera puede probarse”
@sinelaw Interesante. ¿Puede encontrar un contraejemplo para tal afirmación (si es cierto y lo sabemos, entonces se puede probar, pero si es cierto y no lo sabemos, entonces no es necesariamente cierto)?
@alexqwx, Godel tuvo que refutar que... es su primer teorema de incompletitud ( en.wikipedia.org/wiki/G%C3%B6del%27s_incompleteness_theorems )
@sinelaw, OK, proporcione una prueba formal de su declaración.
@sinelaw Creo que te has perdido las condiciones de los teoremas de Goedel. No hizo una afirmación sobre todos los sistemas formales, y si lo hubiera hecho, Goedel habría refutado el teorema de completitud para el cálculo de predicados de primer orden que había probado antes (creo que la axiomización de Hilbert-Bernays de tal).
Voto para cerrar esta pregunta como fuera de tema porque ya hay suficientes teoremas obvios

Respuestas (71)

Teorema (falso):

Uno puede reordenar arbitrariamente los términos en una serie convergente sin cambiar su valor.

¿Tiene un contraejemplo explícito a este hecho?
@DanielRobert-Nicoud (Condicionalmente) serie convergente. =)
Las series ( 1 ) i i es un contraejemplo; vea este artículo de Wikipedia para una discusión.
@MJD ¡Gracias!
@OL Sé (ahora) que reorganizar los términos da un resultado incorrecto; por ejemplo, eso en ( 2 ) = 1 2 en ( 2 ) , pero ¿dónde está la falacia de reordenar los términos?
@alexqwx Aproximadamente, que algunos términos se pueden empujar 'infinitamente lejos', de modo que dos sumas parciales finitas difieren arbitrariamente en muchos términos. Imagine, por ejemplo, tomar los números positivos y reordenarlos como 1, 2, 3, 5, 4, 7, 9, 11, 6, 13, 15, etc. con un número creciente de términos impares entre cada dos términos pares consecutivos; luego, cada número aún aparece en la secuencia eventualmente, pero eso no significa que se conserve ningún concepto como una 'proporción' de impar a par (por ejemplo, densidad natural ).
@DanielRobert-Nicoud: como hay contraejemplos, no es un hecho...
¿Eh? Se pueden reorganizar los términos de cualquier serie, convergente o no. ¿Dejaste algo fuera del enunciado de tu teorema?
@MJD Esa serie no es un contraejemplo de la afirmación "uno puede reorganizar los términos en una serie convergente"; de hecho, el artículo de Wikipedia que citó brinda varios ejemplos explícitos de reordenamientos de esa serie.
En realidad, puede reorganizar los términos en una serie convergente al calcular su valor (o de otro modo). Incluso puede reorganizar los términos sin cambiar el valor, con un poco de cuidado. Lo que no puede es suponer que cualquier reordenamiento preservará la convergencia, o cuando conserva el valor al que convergió.
@alexqwx No hay falacia. Lo que sucede es que la definición habitual de serie se da en términos de una forma muy particular de obtener valores, que no es inmune a la reordenación.
@MarcvanLeeuwen cierto, puedo andar diciendo eso norte = 0 norte = 1 12 , pero para la mayoría de las situaciones esto no es útil y no es cierto.
Eso suena bastante roto, ¿no está eso en contradicción con la conmutatividad de la suma?
@kutschkem Siempre puede realizar una cantidad finita de intercambios de términos (usando la conmutatividad) sin cambiar el valor, pero no puede realizar una cantidad infinita de intercambios; la continuidad no se conserva en un número infinito de operaciones.
@StevenStadnicki Ok, creo que ya veo, ¿puede señalarme algún lugar donde esto se haga explícito (en el artículo de wikipedia vinculado, está produciendo un contraejemplo pero no dice nada sobre "la continuidad no se conserva en un número infinito de operaciones ".
Si una serie no converge absolutamente, hay subsucesiones que van a y . Las personas familiarizadas con el hotel de Hilbert deberían tener claro que las cosas no van a ser invariantes bajo los reordenamientos.
¿No está el problema en cómo analizamos las series, no en las transformaciones inofensivas?
¿Por qué se acepta esta respuesta? Parece que, por la naturaleza de la pregunta, ninguno debería serlo.
@bof claramente significa "Supongamos que tenemos una serie convergente con suma S. Podemos reorganizar los términos arbitrariamente, y la serie aún converge a la suma S"
@MattMcNabb Y mi comentario claramente significa "Sé lo que quisiste decir, pero no lo dijiste".
@Ollie Probablemente sea el que más le gusta al OP. No hay forma de hacer que una pregunta no sea elegible para tener una respuesta aceptada.
Este es particularmente divertido. Si cambia esto a VERDADERO y cambia las reglas de las matemáticas sobre el infinito para permitir este cambio, se cumple el teorema fundamental del cálculo. No puedo probar que el resultado sea completamente consistente, pero parece funcionar bien. Esto también elimina el desagradable mordisco del Axioma de Elección (la duplicación de la esfera ya no se sostiene).
Edité esto para decir lo que realmente significaba. Esto me molestó la última vez que apareció, pero decidí no arreglarlo.
Si esto es cierto, entonces es tan extraño que casi quiero redefinir la convergencia para que sea de otra manera.
Aprendí esto de la manera difícil.
@mathreadler No es necesario redefinir la convergencia; ya existe una convergencia absoluta , que es exactamente lo que se necesita para permitir reordenamientos arbitrarios.

Una forma con volumen finito debe tener un área de superficie finita.

Esto es evidentemente falso. Tome un cilindro de arcilla de diámetro 1. Hágalo rodar en sus manos para que su diámetro sea 1 2 . El volumen es el mismo, pero ahora es cuatro veces más grande que antes, por lo que tiene el doble del área de superficie del cilindro original. (La mitad de la circunferencia, pero cuatro veces la longitud). Enróllalo un poco más y el área de la superficie aumenta de nuevo. Al hacer la serpiente muy larga y delgada, puedes aumentar el área de la superficie hasta el infinito mientras el volumen permanece constante. Esto no solo es obvio, es un lugar común.
El ejemplo familiar de una esponja ilustra otra forma en que un sólido podría tener un volumen finito pero un área de superficie grande o potencialmente infinita.
Esos son buenos ejemplos. Sé que esto no confundiría a nadie capacitado formalmente como la mayoría de los carteles aquí, pero estaba tratando de sugerir un ejemplo de "términos sencillos, la persona promedio" como solicitó OP. Parecía que la mayoría de los ejemplos hasta ahora ni siquiera serían interpretables para una persona no capacitada.
@MJD: su ejemplo de cilindro es bueno, pero solo muestra que el área de superficie puede ser arbitrariamente grande. El OP señala que el área de la superficie puede ser literalmente infinita.
Sí, pero para que sea realmente infinito, sigues el mismo principio y usas una serpiente que es gruesa en un extremo pero se vuelve más delgada a medida que avanzas hacia el infinito. Entonces lo único sorprendente es que esta serpiente infinitamente larga pueda tener un volumen finito. Pero nos tragamos esa rareza hace mucho tiempo; Lo mencioné en otra parte de este hilo junto con la paradoja de Zeno.
Es decir, no haces rodar toda la serpiente, sino solo la mitad derecha en cada paso.
@MJD: Si continúa expandiendo el área de la superficie de la arcilla, ¿no llegará a un punto en el que la distancia entre los átomos sea lo suficientemente grande como para que la arcilla se desintegre? Si ese fuera el caso, no deberías poder expandir infinitamente el área de la superficie de una arcilla (no sé, no soy matemático).
Yo tampoco sé, soy un ignorante en física. ¿Estás seguro de que la arcilla está hecha de átomos?
Matemáticamente esto puede ser cierto, pero físicamente imposible, ya que la materia es discreta. El límite sería una fila de átomos.
Esponja Menger! 0 Volumen ¡Superficie infinita!
Puede establecer un límite superior si restringe la forma para que sea convexa.
"Una forma con volumen finito tiene un área de superficie bien definida (finita o infinita)" es menos ambicioso, pero probablemente también sea falso (a menos que el término forma tenga un significado restringido que desconozco aquí).
En la misma línea, pero posiblemente menos apreciado, un objeto de masa finita puede fallar (matemáticamente hablando) para tener un centroide finito.
De manera similar para 2D, "una figura con área finita debe tener un perímetro finito". Una interpretación del conjunto de Mandelbrot es prueba de que es falso: área finita, perímetro infinito.
No soy matemático, ni siquiera cerca. Pero, ¿no podría probarse esto a partir de la relación V/A? En el caso del cilindro creo que es h/(h+r).
No es bueno para las matemáticas o la física, pero incluso si tomamos un cilindro puramente matemático, si lo estiramos hasta el infinito para demostrar que su área de superficie es así, ¿no se vuelve infinito el volumen exactamente en el mismo instante?
Tetraedro de Koch . Ahí está tu prueba.
@ deed02392 No, no es así, pero esta pregunta es exactamente por qué el 'contraejemplo de arcilla' no es obvio.
Me entristece que mi respuesta mejor calificada hasta la fecha sea una afirmación que es completamente falsa.
Todavía no entiendo cómo esta afirmación es falsa. Dado que el 'infinito' no existe y es solo una construcción matemática para ayudar a nuestra comprensión de la naturaleza, me parece que cualquier cosa finita es de hecho finita.
El Cuerno de Gabriel también es un ejemplo. Volumen de π con una superficie infinita.
@user3932000: Lo bueno del Cuerno de Gabriel es que tiene una forma tan simple, un sólido de rotación que tiene, como tal, secciones transversales convexas en planos apropiados. Eso lo hace mucho más sorprendente que cosas como esponjas o cilindros estirados.
Esto es como argumentar que una línea recta tiene un área o un plano tiene un volumen.

Suponer A y B están jugando el siguiente juego: A elige dos números diferentes, a través de algún método desconocido para B , las escribe en tiras de papel y las mete en un sombrero. B saca una de las tiras al azar y examina su número. Luego predice si es el mayor de los dos números.

Si B simplemente lanza una moneda al aire para decidir su predicción, acertará la mitad de las veces.

Obviamente, no hay ningún método que pueda hacerlo mejor que el lanzamiento de una moneda.

Pero existe un método de este tipo, descrito en Thomas M. Cover " Elija el número más grande " ( Problemas abiertos en comunicación y computación Springer-Verlag, 1987, p152), que describí brevemente aquí , y en detalle aquí .

¿Existe realmente un concepto consistente de una distribución uniforme de norte o R + ? Podría entender una distribución como mi X produciendo un número real aleatorio a partir de 0 a , pero ¿qué pasa con una distribución uniforme?
¿Quién dijo algo sobre una distribución uniforme en norte ? Ciertamente yo no.
Entonces, ¿la distribución que la persona A usa para elegir no se especifica?
Sí. Eso era lo que pretendía al decir que el método era desconocido para B , que es todo lo que es importante aquí. ¿Cómo podría hacer esto más claro?
Lo siento, no debería abordar esto aquí, leeré tu enlace lo mejor que pueda y comentaré tu otra publicación si es necesario, ya que creo que es un mejor lugar para la discusión. Estoy de acuerdo, dos personas que juegan este juego, si les preguntan "¿hay una mejor manera que lanzar una moneda?" ciertamente dirían que no, así que +1 en el supuesto de que su resultado es correcto ^_^ Supongo que tengo problemas para asociar mecánica procesos a conceptos no computables (como seleccionar un número real aleatorio).
Esto podría hacerse completamente computable: puede generar números computables dentro de una tolerancia arbitraria de acuerdo con una distribución.
Si, absolutamente. Es una estrategia real que uno puede ejecutar y predecir correctamente con una probabilidad mayor que 1 2 . Me esforcé un poco en explicar esto en mi publicación original, incluyendo, por ejemplo, "los métodos para hacer esto están bien estudiados" y tal con la esperanza de persuadir a los dudosos.
Creo que esta es hasta ahora la mejor respuesta en este hilo. +1 a esta respuesta y su respuesta anterior que explicaba la paradoja.
La persona A puede elegir muy fácilmente una estrategia que mantenga las probabilidades de que la persona B acierte muy cerca de 1/2 (lo suficientemente cerca como para que no valga la pena el esfuerzo de usar esta estrategia en la práctica contra un oponente informado), pero la probabilidad es mayor. de 1/2 sin embargo. +1 para ti.
Ese es el caso. Describí tal estrategia por parte de la persona A en mi publicación original . Pero también creo que es interesante notar que A no puede garantizar el límite B la probabilidad de éxito de ( 1 2 , 1 2 + ϵ ) para cualquier ϵ menos que 1 2 .
@DanielV: "la distribución que usa la persona A" es un concepto que es ajeno al problema en cuestión, y si entiendo correctamente, simplemente mencionar esta frase podría ser una fuente de confusión. No hay distribución ; la afirmación involucra la cuantificación universal sobre todas las elecciones de A. Es decir, si A elige 1 y 7, esto funciona. Si A elige -sqrt(2) y 763.45, esto funciona. Etcétera.
No creo que sea obvio que no exista tal estrategia, de hecho creo que la mayoría de los legos la aplicarían intuitivamente (en una forma heurística, es decir, "¿suena grande ese número?" en el sentido de "¿habría sido probable que hubiera elegido dos números y encuentra que este es el más grande"). Lo que es obvio, y de hecho correcto, es que la estrategia puede ser subvertida fácilmente por la persona A, eligiendo ambos números muy grandes y cercanos entre sí.
Por si no lo crees: jsfiddle.net/GeB8V o consola estofunction pickPos() { return Math.floor(Math.random() * Number.MAX_SAFE_INTEGER); }; function play() { var y = pickPos(); x = pickPos(); z = pickPos(); return ((x < y && x < z) || (x > y && x > z)) ? true : false }; counter = {wins: 0, losses:0}; i = 100000; while(i--) { counter[play() ? 'wins' : 'losses']++ }; counter
@qwertymk Esto supone que la persona B conoce la distribución de probabilidad de la persona A.
@qwertymk También asume un límite superior en los números que se eligen.
@DanielV + Tyilo Aparte de esos puntos, todavía es bastante sorprendente que funcione
Esto se puede explicar de forma muy sencilla. B adivina la estrategia de A. Si se equivoca, sus probabilidades son 1/2. Si tiene razón, sus probabilidades son mejores que 1/2. Siempre que sus probabilidades de adivinar la estrategia de A sean mejores que 0, sus probabilidades de ganar son mejores que 1/2.
El elemento adicional es que si convertimos esto en un juego iterativo, B necesita cambiar su suposición de una manera que A no pueda predecir con 100% de certeza. En resumen, dado que la predicción de B sobre A siempre tiene la posibilidad de ser correcta, y la predicción de A sobre la predicción de B siempre tiene la posibilidad de ser incorrecta, B siempre está por delante.
@AleksandrDubinsky Creo que estás equivocado. El método real de A es elegir entre N(0,1). B supone que el método de A es elegir entre N(3,1). B perderá mucho más del 50%.
@MatthewFinlay En ese caso, B casi siempre dirá que eligió el más pequeño. Todavía tiene un 50 % de posibilidades de elegir el más pequeño.
@AD si A tiene un continuo de estrategias (por ejemplo, distribuciones de probabilidad), ¿cómo puede B "acertar" con probabilidad? > 0 ?
Yo puedo haber estado equivocado. Hay algunas conjeturas que pueden arruinar las posibilidades de B. Por ejemplo, la estrategia de A es hacer que el número alto sea par y el bajo impar. B adivina que la estrategia de A es la inversa y siempre pierde. Si promediamos la posibilidad de que A decida alto = par y él elija alto = impar, B vuelve a 50/50. En algunas estrategias de adivinanza, equivocarse duele. La estrategia de adivinanzas propuesta por MJD es excelente porque nunca puede perjudicar a B. Me pregunto si es único, o si hay otros con esta propiedad.
Por cierto @MJD, gracias por publicar esto; ¡Es uno de mis problemas de probabilidad favoritos!
@Goos Hay un continuo de números reales. Eso no impide que B adivine la mediana de la distribución de A (que es como interpreto la solución oficial). Por ejemplo, si asumimos que la distribución de A es un rango pequeño en cualquier parte de la recta numérica real, B todavía tiene una probabilidad distinta de cero de adivinar un número que se encuentra dentro de ella (y ganar).
@Goos Ok, entiendo. El truco de la prueba es que entre dos números reales, hay un racional. Es el racional que se puede adivinar con probabilidad distinta de cero.
@Goos ¿Estás seguro? Adivinar un racional es como adivinar dos números enteros. Si no puedo adivinar un racional, no puedo adivinar un número entero. Si no puedo adivinar un número entero, no puedo adivinar un número entero que se encuentre dentro de un rango. ¿Cómo es que puedo adivinar un real dentro del mismo rango? (Trabajando a la inversa: si puedo adivinar un real entre dos reales, entonces para adivinar un m/n racional solo necesito adivinar dos reales en rangos (m,m+1) y (n,n+1) y redondear hacia abajo .)
Evite discusiones extensas en los comentarios.
Existe un método para generar los números para los que no puede funcionar mejor que 1/2.
@Joshua Eso es falso. ¿Puedes explicar tu supuesto método?
Al volver a introducir la métrica citada, podemos producir trivialmente un patrón de selección que rompa esa métrica. Por supuesto, hay un contra-patrón para derrotar a ese. Elegir aleatoriamente entre las estrategias L1 y L2 debería resultar en 1/2.
@Joshua Estás equivocado. Incluso si X saber eso Y está eligiendo de una distribución normal, no hay nada X puede hacer para reducir Y la probabilidad de éxito de 1 2 .
Tirar una moneda al aire es una pista falsa; si quiero una estrategia en la que mi probabilidad de ganar sea 1/2, entonces simplemente puedo quedarme con la carta que elegí inicialmente (o siempre cambiar). Para hacerlo mejor que esto, sólo tengo que darme algún incentivo para preferir mantener un número mayor. La estrategia mejorada más simple es mantener un número positivo y cambiar si obtengo un número negativo (aunque podría compararlo con un número que no sea 0). …
… Qué tan bien funcione esto depende del método de A para elegir los números. Si A siempre elige dos números positivos, esto equivale a mantener siempre mi número, por lo que no hay mejora. De manera similar, si A siempre elige dos números negativos. Pero si A siempre elige un número positivo y un número negativo, entonces esta estrategia es perfecta y siempre gano. Para resolver el problema, necesito aumentar mi probabilidad por encima de 1/2 sin importar lo que haga A, por lo que aún no es una solución, pero debería ser suficiente para desterrar la idea de que no es posible mejorar. …
… Para obtener una solución completa, todo lo que tengo que hacer es elegir mi punto de corte al azar. Dado que A podría limitar los números a cualquier rango limitado, posiblemente bastante grande (o pequeño) y angosto, necesito una distribución de probabilidad que asigne probabilidad positiva a cualquier intervalo de números no trivial. Pero hay muchas distribuciones como esa, así que elija una, utilícela para seleccionar al azar un punto de corte y cambie si mi número está por debajo del punto de corte. Y esa es la solución descrita por el OP.

Esto es elemental en comparación con la mayoría de los otros ejemplos, pero ¿qué tal

Hay más números racionales que enteros.

La belleza reside en su sencillez. La falsedad de esta una de esas observaciones muy elementales, la parte de mi cerebro que está impulsada por el sentido común todavía se niega a aceptar.
Mi cerebro en general se niega a aceptar esto. :) Parece que incluso si restringes b a { 1 , a + 1 } , a / b te da el doble de valores que a .
Y hay la mitad de enteros impares que enteros (pares o impares).
No lo sabía, ¿podría explicar qué está mal con la siguiente prueba (falaz)? ( norte norte ) ( metro 1 = norte , metro 2 = norte + 1 2 ) R , lo que debería significar que por cada número entero, existen al menos dos números reales. Creo que el problema está en la infinitud de los números naturales, y es un poco como decir 2 > , lo que obviamente es incorrecto. ¿Es esa la trampa?
@Jori: No he estudiado las pruebas y la terminología en años, pero creo que una forma algo simple (¿simplista?) De decirlo es que la existencia de un mapeo de inyección (no uno a uno) no excluye la existencia de un mapeo uno a uno. PD Algunas personas podrían considerar que 2∞>∞ es obviamente cierto .
@Jori: el teorema establece números racionales , no números reales . Y la prueba clásica es, afaik, (diagonalización de Cantor)[ en.wikipedia.org/wiki/Cantor's_diagonal_argument] . Puede idear una estrategia que enumere los números racionales de manera determinista. Esto asigna un número natural a cada número racional, mostrando que los conjuntos tienen el mismo tamaño.
Ups, malinterpretado. Buscaré en tu enlace. ¡Gracias! ¿Cuenta también para los números reales?
@Jori: (1) Debería haber dicho "la existencia de un mapeo de inyección no sobreyectivo (no una correspondencia uno a uno) entre dos conjuntos transfinitos (es decir, infinitos) no excluye la existencia de un uno a - una correspondencia. Obviamente, la existencia de un mapeo de inyección no sobreyectivo entre dos conjuntos finitos excluye la existencia de una correspondencia uno a uno. (¿O es otra verdad "obvia" que no es realmente cierta?)
@Jori: (2) ¡ No! Como dio a entender Cornelius en su comentario, el argumento diagonal de Cantor ( enlace correcto ) muestra que hay conjuntos infinitos (como el conjunto de números reales) que no se pueden poner en correspondencia biunívoca con el conjunto (infinito) de números naturales. Hay (¡infinitos!) cardenales transfinitos diferentes . En pocas palabras, no todos los infinitos son iguales.
@gnasher729: Seguramente no es tanto falso como sin sentido o verdadero decir que hay la mitad de enteros impares que enteros. Lo sorprendente es que 2 0 = 0 .

Sigo insistiendo en esto, porque creo que es un ejemplo espectacular de algo que se puede demostrar que es completamente obvio (no solo porque lo parece, sino porque se creyó mucho durante tanto tiempo) y, sin embargo, es completamente incorrecto:

Suponer Φ es una propiedad que puede o no tener algún objeto. Luego hay una colección. S Φ de todos los objetos con propiedad Φ .

Muchos matemáticos serios e incluso famosos siguieron adelante con este principio intuitivamente obvio pero completamente falso, cuya demolición sacudió las matemáticas hasta sus cimientos y marca el comienzo de la lógica moderna y la teoría de conjuntos.

(Hay muchos contraejemplos, de los cuales el más conocido es Φ ( X ) = X no es miembro de la colección X ”. Para otros, ver ¿Es la paradoja de Russell la única contradicción posible al esquema axiomático de comprensión debido a Frege (1893)? { X : PAG ( X ) } y la paradoja de la comprensión general en la teoría de conjuntos, además de la paradoja de Russell ).

+1 para un gran ejemplo, pero creo que el uso de "propiedades", "objetos" y "colecciones" quita el impacto al ser impreciso. Definitivamente hay teorías de conjuntos consistentes donde hay colecciones que no son objetos y la declaración es verdadera en algún sentido. Expresarlo usando la palabra conjuntos y en el contexto de la teoría de conjuntos ingenua tradicional y las formalizaciones modernas ampliamente utilizadas de la teoría de conjuntos mejoraría esta respuesta en mi opinión.
¿Por qué está mal?
No estoy de acuerdo, creo que esto funciona bastante bien con términos indefinidos siempre y cuando tengas una noción de contención, no necesitas nada más.
Joshua tiene razón aquí, y este resultado contrario a la intuición se aplica en muchas situaciones relacionadas. Por ejemplo, WV Quine dice: “No se puede negar que este principio se usa constantemente, tácitamente, cuando hablamos de los adjetivos como verdaderos de las cosas: el adjetivo 'rojo' es verdadero de una cosa si y solo si la cosa es roja, y correspondientemente para todos los adjetivos… Es un principio difícil de desconfiar y, sin embargo… [debe] ser abandonado o al menos restringido de alguna manera”. De manera similar, se podría pensar que sería posible construir un catálogo C Φ de todos los libros con alguna propiedad Φ , pero no lo es.
@mattecapu No hay colección S Φ de todas las colecciones que no son miembros de sí mismas.
Oh, veo tu punto
He comenzado a encontrar que esta formulación del problema es engañosa. Uno siempre puede considerar una "colección" de objetos que satisfacen una fórmula, ya que uno puede considerarlos individualmente y nocionalmente, los que lo hacen están separados de los que no lo hacen. El problema es precisamente qué es un "objeto", y qué colecciones de ellos forman un "conjunto". En ZF, son lo mismo, pero las colecciones definidas por fórmulas arbitrarias pueden no serlo; esto es diferente a decir que no existen: más bien, no están sujetos a los axiomas como "objetos".
Realmente creo que ese no es precisamente el problema. El problema aquí no tiene nada que ver con ZF, con axiomas, o con cómo definimos exactamente "objetos"; es un problema fundamental con la noción intuitiva de lo que significa que las cosas tengan propiedades. ¿Qué le parece esta formulación: “Para cualquier propiedad Ψ , se puede construir un catálogo que enumere todos los libros con propiedad Ψ ?” Y la respuesta es que para algunas propiedades Ψ , simplemente no puedes. ¿Es eso lo suficientemente concreto y no teórico?
Wow, gracias. He oído hablar de la paradoja de Bertrand, pero siempre pensé en ella como un ejemplo artificial que demostraba una falla en el pensamiento actual de la época. Verlo como un contraejemplo a esto realmente demuestra cuán aterrorizada debe haber estado la comunidad matemática.
Me inclino a pensar que tener un catálogo de cosas rojas no es un problema porque la definición no es recursiva. ¿Puede dar una referencia a una discusión que demuestre cuándo esta intuición es peligrosa?
@Polymer Esa es, de hecho, una forma en que las personas han tratado de resolver este problema: la palabra mágica para este tipo de restricción (que cada conjunto solo se refiere a una 'clase inferior' de conjuntos) es teoría de tipos . El problema es que hay muchas construcciones matemáticas que son muy difíciles de hacer con "seguridad de tipos".
No, esa definición es circular.
@MJD No importa cómo lo diga usando términos vernáculos, implícitamente me está llevando a considerar una "colección" como un tipo de "objeto" o un "catálogo" como un tipo de "libro". Eso no es necesario, ni obvio; de hecho, la afirmación de que es verdadera es exactamente el contenido del axioma de especificación (bajo las circunstancias apropiadas). Nocionalmente, los objetos que satisfacen la fórmula pueden considerarse como un todo, pero ese todo no es necesariamente un objeto del tipo descrito por los axiomas. (IOW: no, un catálogo no es un libro, por lo que su reformulación no me convence).
"cuya demolición sacudió las matemáticas hasta sus cimientos y marca el comienzo de la lógica moderna y la teoría de conjuntos" ¿En qué parte de la historia de la teoría de conjuntos entra esto?
Esta es la paradoja de Russell , descubierta en 1901, que destruyó Grundgesetze der Arithmetik (1903) de Frege y trabajos anteriores. Las respuestas inmediatas a la paradoja incluyen Principia Mathematica de Whitehead y Russell (1910) y el trabajo de Zermelo sobre la teoría axiomática de conjuntos a partir de 1905, que finalmente se convirtió en ZFC, que domina la teoría de conjuntos hasta el día de hoy.
¿La paradoja requiere que una colección cuente como un objeto? ¿Qué pasa si la propiedad es simplemente "el libro no está en un catálogo"? No se puede hacer un catálogo de libros sin catalogar. En pocas palabras, si permite que las definiciones (de propiedades, o lo que sea) sean lo suficientemente sofisticadas como para que estén completas (o simplemente circulares), entonces no puede esperar que se detengan siempre o incluso saber si se detendrán. .
¿Sería el equivalente lógico "Esta declaración es falsa"?
No, eso no es equivalente en absoluto. ¿Por qué pensarías que sería?
@Jab el "equivalente lógico" es
definir  PAG ( q ) := ¬ q ( q )
en la superficie parece no recursivo, pero ¿qué pasa con P(P)...
@MJD Del caso de falla, quise decir.
No veo cuál es la diferencia. La teoría ingenua de conjuntos asumía que cualquier conjunto que pueda definirse debe existir. Es como decir que cualquier enunciado que se pueda construir debe ser verdadero o falso. "Esta declaración es falsa" no es ni verdadero ni falso.
Me pareció ver una declaración como esa en Principia Mathematica de Russel & Whitehead , y supuse que su sistema de tipos aseguraba que no hubiera contradicciones.
Creo que "Esta declaración es falsa" tiene un grado de notoriedad injusto en comparación con "Esta declaración es verdadera", lo que causa muchos más problemas en el mundo real.
@R..: No estoy de acuerdo; se necesita el uso de un lenguaje no matemático para que a la persona promedio le sorprenda que es falso.

En un hilo relacionado con mathOverflow , Gowers señaló la siguiente afirmación obvia pero falsa:

Dejar I 1 , I 2 , ser subintervalos de [ 0 , 1 ] cuya longitud total es estrictamente menor que 1. Entonces la unión de los I i no puede contener q [ 0 , 1 ] .

(Tenga en cuenta que si q se reemplaza con R , la afirmación es verdadera.)

Encuentro el hecho de que todos q puede ser cubierto por una familia arbitrariamente pequeña de intervalos para ser uno de los más extrañamente contradictorios en todas las matemáticas.

¡Interesante! ¿Hay un ejemplo explícito de tal unión o solo se puede demostrar que existe?
Tu ordenas los racionales q 1 , q 2 , , q norte , y coloca un intervalo de ( ϵ 2 ) norte alrededor de cada uno, eligiendo ϵ ser menos que 1 .
Por supuesto, el teorema sigue siendo cierto si tenemos un número finito de intervalos; uno necesita la construcción infinita para hacer este trabajo, que es probablemente donde se encuentra la brecha en la intuición.
@dfan, ¿funciona esto para cualquier e que sea menor que 1? Y por ordenar los racionales, ¿te refieres a ordenar los racionales que están en [0, 1]? La construcción todavía se siente mal para mí. Lo expresaste de una manera que sugiere que el orden de las proporciones es arbitrario, pero si el primer racional era 0,00000001 para e = 0,9, entonces el intervalo estará muy lejos de [0,1]. ¿Que me estoy perdiendo aqui?
Aparte, el no teorema anterior introduce un concepto importante para comprender la integración. enlace
@Cruncher Sí, funciona para cualquier ϵ < 1 desde i = 1 ( ϵ 2 ) norte < 1 . Y sí, se está refiriendo a los racionales en [ 0 , 1 ] aunque si piensas en el argumento, verás que no es estrictamente necesario, podría cubrir todos los racionales con longitud finita. Finalmente, la afirmación no era que los intervalos serían iguales a q [ 0 , 1 ] , sino que lo contienen.
@cruncher Primero debe convertir los racionales en una lista, digamos 1 1 ; 1 2 , 2 1 ; 1 3 , 2 2 , 3 1 ; 1 4 , 2 3 , 3 2 , 4 1 ; . Luego alrededor de la norte th racional en la lista, pones un intervalo de ancho ϵ 2 norte . Es decir, si el norte lo racional es q , entonces el norte el intervalo es ( q ϵ 2 ( norte + 1 ) , q + ϵ 2 ( norte + 1 ) ) , que ciertamente incluye q , y que tiene una longitud ϵ 2 norte . y puedes tener ϵ ser tan pequeño como quieras. Y, sin embargo, cada racional está contenido en uno de estos intervalos.
Hace [ a , a ] cuenta como un intervalo? Entonces el I i puede ser simplemente un ordenamiento de los números racionales en [ 0 , 1 ] .
@alex.jordan: no, el requisito suele ser intervalos abiertos o intervalos de medida positiva.
@Cruncher Tiene razón, eso es un problema porque dice "subintervalos" en la respuesta. Entonces, dos cosas: 1) Creo que la construcción se puede arreglar para que nuestros intervalos salgan de [ 0 , 1 ] ; por ejemplo (esto necesita un poco de verificación, sin embargo, podría ser imposible hacerlo) si algunos de sus subintervalos se encuentran fuera [ 0 , 1 ] , solo sigue encogiéndote ϵ hasta que cada subintervalo esté en [ 0 , 1 ] . 2) Incluso si no puede hacer eso, es sorprendente que pueda cubrir los racionales con intervalos cuya suma es arbitrariamente pequeña, ya sea que pueda encajar todos los subintervalos en [ 0 , 1 ] O no.
@Cruncher Nos dice que los racionales son realmente insignificantes cuando intentas medir la longitud; por ejemplo, se puede decir que la longitud de [ 0 , 1 ] q = 1 . Si está más interesado, lo que debe buscar en Google es "medida Lebesgue". Los racionales tienen medida de Lebesgue 0 , y esta es la forma estándar de mostrarlo.
@Cruncher Numberphile tiene un video interesante en youtube.com/watch?v=aIggWlKr41w
@Polymer: Correcto, el concepto de un conjunto de medida cero.
@Ovi: los subintervalos no necesitan estar centrados en los racionales, por lo que no necesita reducir ε, solo tome la intersección con [0,1].
Esta no es una familia de intervalos arbitrariamente pequeña, sino una familia de intervalos con una medida total arbitrariamente pequeña.
Este problema fue discutido en un video de 3Blue1Brown sobre teoría musical.

Si una función F ( X ) tiene una asíntota horizontal, entonces límite F ( X ) = 0

+1 Me encanta este! Un contraejemplo es pecado X 2 X .
Una condición suficiente para que esto sea cierto es que la segunda derivada esté acotada.
@AP o incluso solo eso límite F ( X ) existe
Como estudiante universitario intenté escribir un ε - d prueba de que si F ( a ) > 0 entonces hay algún intervalo abierto que contiene a en la que F esta incrementando. Y pensé que mi habilidad para escribir ε - d pruebas faltaba algo porque no podía averiguar cómo hacerlo.
Ah, tenemos respuestas casi duplicadas. Lo interesante es que puedes agregar las siguientes restricciones para hacerlo aún más extraño: F es C , acotado y monótono.
@Kaj Hansen: ¿Puede mostrarme un ejemplo? F ? :)
Aunque creo que puedo imaginármelo, en realidad: piensa en un F que tiene escalones suaves, pero cada vez más empinados y cada vez más cortos cuando se grafica. Entonces el límite no existe. (¿Por qué?) Pero ¿qué pasa con uno donde el límite existe?
Creo que lo que describes en realidad puede funcionar con algunos retoques. Vea mi respuesta de 10 votos a favor a continuación.
@ mike4ty4 Si el límite existe, entonces debe ser igual a 0.
¿Sigue siendo cierta esta afirmación si F ( X ) no se le permite cruzar su propia asíntota horizontal? Porque creo que de ahí es de donde podría provenir la falsa intuición.
@Ryan ¿Quieres decir si F ( X ) es monótono? Sí, incluso en este caso no podemos decir que el límite es igual 0 , como otros han señalado
Una definición muy extendida entre los analfabetos es que una asíntota es una línea a la que la gráfica de una función se va acercando pero nunca toca.
@Ryan Si tomas F ( X ) = pecado ( X 2 ) + 2 X , entonces F ( X ) es eventualmente positivo, pero la derivada aún no tiene límite.
@MichaelHardy re "analfabetos": ¿Está objetando la parte "nunca toca" o la vaguedad de la definición? Si es lo primero, entonces eso es un poco duro dado que es literalmente (¡ja!) La etimología de la palabra.
Principalmente la parte "nunca toca": le dirán que si una curva cruza o toca una línea, entonces esa línea, por definición, no es una asíntota de la curva. Pero también, la función X y = pecado X X tiene y = 0 como una asíntota, pero no sólo sigue volviendo a cruzar esa línea, sino que tampoco sigue acercándose, ni siquiera "eventualmente".
Por desgracia, he utilizado constantemente este "hecho" para encontrar soluciones de equilibrio a las ecuaciones diferenciales. Ahora todo mi trabajo está en peligro.

0. 9 ¯ < 1

Probablemente el más famoso de los "obvios" pero falsos.

Me pregunto cuán "obvio" es esto entre las personas que saben cómo se definen realmente los números reales.
La pregunta planteaba lo que la persona promedio consideraría cierto.
Otra forma de afirmar esto es "cada número real se puede escribir como una expansión decimal infinitamente larga exactamente de una manera", para que sea más obvio para la mayoría de las personas.
Este es, con mucho, el mejor en la página imo. Algunos de los otros son tan complejos para la persona promedio que no serían "obviamente" verdaderos para casi nadie.
En números reales es falso, pero es cierto para los hiperreales. Es solo por convención que usamos por defecto la teoría de los números reales en lugar de algún otro sistema.
No consideraré ningún tipo de número 0.999... directamente es un número, pero este "obvio" es realmente cierto, déjame explicarte. 0.999... es igual a 1 - 1/∞. Lo que hace un extraño tipo de infinito que es infinitamente cercano a uno.
Pero 0.999... no es igual a 1 - 1/∞ (a menos que por 1 - 1/∞ solo quieras decir 0).
Creí esto hasta que fui 9. 9 ¯ años. En mi 10 el cumpleaños, vi la luz. :-) ⁠
Recuerda que una cadena infinita de 0 s que termina con un 1 nunca termina en un 1 , porque eso contradiría la idea de que el infinito es interminable

la falsedad de

Dejar S Sea una familia infinita de números estrictamente positivos. Entonces S =

ha estado alucinando a la gente durante miles de años. Es la base de la paradoja de Zeno, pero si crees que la paradoja de Zeno es vieja y cansada, considera que también es la base de la paradoja del Cuerno de Gabriel (también mencionada en este hilo), que todavía desconcierta a la gente.

Este es, en mi opinión, el mejor ejemplo aquí. Es fácil de entender para un profano con una pequeña explicación y, a primera vista, parecería increíblemente obvio que si sigues sumando números, siempre se hará más grande.
@ jpmc26 Siempre se hace más grande a medida que sigues agregando números; simplemente no lo hace necesariamente sin límite.
Por otro lado, si la familia es incontable, se cumple.
Acabo de leer este y los comentarios... y es realmente raro. (Tengo una maestría en física, por lo que no estoy completamente familiarizado con este tipo de cosas).
Siento que esto no es tan loco, aunque tal vez sea porque tengo experiencia, pero quiero decir 1.1111 = 1 + 0.1 + 0.01 + .
@Almo Lo más probable es que hayas estudiado esto en tus cursos básicos de cálculo. Es solo una suma infinita, y algunas sumas infinitas convergen en un valor en lugar de divergir. ¿Recuerdas todas esas pruebas de convergencia/divergencia de series en la época en que estudiaste las integrales por primera vez?
¡Ooooh! Lo leo como " enteros estrictamente positivos " que creo que divergirían. ¿Bien? (Demasiada programación)
@Almo La suma de una serie infinita de enteros positivos necesariamente diverge al infinito, sí (no puede ser menor que la suma de una serie infinita de 1).
@user87690 ¿Qué pasa si s S , X R + 0 : s < X ; eso es, s es infinitesimal?
@WChargin: Entonces supongo que no se vuelve a sostener ya que la suma finita de infinitesimales es infinitesimal, por lo que cualquier subsuma finita es menor que decir 1 . Para mi comentario anterior, asumí que el número estrictamente positivo es un número real.
@ user87690 No creo que sea correcto. La suma de un número incontable de infinitesimales positivos no convergerá, por el principio de transferencia (ya que no converge para ningún número real positivo). Alternativamente, no converge porque eventualmente sumará más de 1 después de llegar a un número hiperentero de términos (todavía menos de ω ).
@MarioCarneiro No digo que converge, solo digo que no es ∞. Para que quede claro, la suma de la que hablo se define como límite de la red de subsumas finitas. ¿No es cierto que cualquier suma finita de infinitesimales es infinitesimal por inducción ordinaria?
@ user87690 Puedo estar equivocado, pero creo que esto es falso, en el sentido de que si intenta escribir lo que realmente quiere decir con "finito", hará referencia explícita a números no estándar o se verá obligado a permitir números naturales no estándar ser también "finito". Para simplificar, considere en cambio una suma contable de números iguales (la suma incontable se reduce a este caso). Para cualquier número positivo ϵ hay una serie de terminos norte tal que la suma es mayor que 1 (y multiplicando esto por METRO puedes hacerlo tan grande como quieras). (continuación)
... Por el principio de transferencia esto también es cierto para infinitesimales ϵ , aunque ahora norte es también un número natural no estándar (pero aún "finito"). En otras palabras, si agregas una gazillionésima a sí misma una tropecientos veces, todavía obtienes una. Si acepta la existencia de números infinitesimales, se ve obligado a aceptar también hiperenteros, por lo que la suma todavía diverge a .
La declaración que está tratando de hacer es que una suma de norte números infinitesimales donde norte Este estándar también es infinitesimal, pero no es posible establecer esto dentro del modelo. (Alternativamente, podría decir que la suma de 2, 3 y 4 infinitesimales es infinitesimal, pero no para general norte , ya que también es cierto que para cualquier infinitesimal ϵ hay un norte tal que norte ϵ > 1 .)
@ user87690 Hay integrales finitas 1 y , y > 0 así que esperaría que una cantidad incontable de valores positivos aún pudiera tener una suma finita.
@MarkHurd S norte := { X S : X > 1 / norte } . Si algun S norte es infinito, entonces claramente S = . Por otro lado, si todos ellos son finitos, entonces S ya que su unión es contable.
@MarioCarneiro Lo entiendo, tienes razón. Pero no estaba tratando de afirmar nada dentro del modelo. Solo tenemos objetos. ω , R con incrustación elemental a algunos R . Los infinitesimales son ciertos miembros de R , estamos sumando R y la suma infinita es un límite de la red de subsumas finitas donde finito significa biyectiva a un miembro de ω .
Zeno se asocia con 9 sobrevivientes a 40 argumentos atribuidos. Todas las referencias. Es propenso a errores asociarlo con una variante de lo que dice esta respuesta. Sin embargo, cualquier persona interesada en validar dicha asociación afirmada puede encontrar útil el artículo en mathpages.com/Reflexiones sobre la relatividad de Kevin Brown . Intenta presentar lo que realmente se sabe sobre el trabajo de Zeno y mostrar que las 2 paradojas más famosas " lejos de estar viciadas por la convergencia de series infinitas, en realidad dependen de la convergencia de series geométricas ".
@ usuario87690 Aceptado, y la falla con mi declaración es el nivel establecido (que es lo que su S norte are) de una integral son todos infinitos, pero todos infinitesimales.
es interesante que S = y 1 / norte = C ; F i norte i t mi ambos están en esta lista. Entonces, ¿qué tal "si algo es obviamente cierto, no puede ser obviamente falso" como pag o s i t i v mi = claramente es ambos.

Cada cadena de subconjuntos de norte es contable.

Hasta ahora encuentro que esto es lo más cierto para mí. Por mi vida, todavía no puedo darme una buena metáfora que la haga obviamente falsa, aunque conozco un contraejemplo matemático. Otras respuestas, o nunca las encontró obvias, o si lo hice, puedo corregir rápidamente el concepto erróneo mediante una forma alternativa de verlo. Probablemente debería agregar un contraejemplo para este en la respuesta.
Por "cadena", ¿nos referimos a una colección de subconjuntos de N que está totalmente ordenada por la relación de subconjunto? Puedes representar el intervalo [ 0 , 1 ] como tal cadena. para cada número X en ese intervalo, el subconjunto contiene (redondeando hacia abajo) el primer 9 X números de un dígito, el primero 90 X números de dos dígitos, el primero 900 X números de tres cifras, etc.
@TannerSwett Ese es el significado, y el tuyo es un buen ejemplo.
@TannerSwett Más directamente, considere los cortes de números racionales de Dedekind.
@DustanLevenstein Claro. o, deja Z [ i ] sea ​​el conjunto de todos los enteros gaussianos, y sea S α = { z Z [ i ] : 0 < a r gramo ( z ) < α } .
La prueba apareció en algunas publicaciones en este sitio, por ejemplo aquí y aquí .

La conjetura de Keller es obviamente cierta:

Dejar R norte estar completamente cubierto con idéntico, no superpuesto norte -cubitos. Debe haber dos cubos que compartan una cara.

(Por ejemplo, cuando norte = 2 cubrimos el plano con pequeños mosaicos cuadrados, y la conjetura establece que debe haber dos mosaicos que comparten un borde. Esto es cierto.)

Sin embargo, la conjetura es falsa para todos norte > 7 .

No estoy seguro de comprar este ejemplo yo mismo, porque no estoy seguro de que algo relacionado con cualquier cosa de 7 dimensiones pueda considerarse intuitivamente obvio.
¿No sería el hecho de que sea cierto para norte = 1 , 2 , 3 (y norte = 4 , 5 , 6 ) lo llevan a sospechar que podría ser cierto para todos norte ?
¿Qué significa "cara" incluso en 7 dimensiones? ¿Es una cosa de 6 dimensiones o una cosa de 2 dimensiones?
¿Qué queremos decir exactamente con "compartir una cara" en este contexto?
Es de 6 dimensiones: el hipercubo es [ 0 , 1 ] 7 y obtienes una cara fijando una coordenada para que sea 0 o 1 .
¿Qué quieres decir con "mosaico"?
por favor dé una referencia para este hecho.
Sin más calificación, asumiría que mosaico significaba en una cuadrícula n, lo que estoy seguro significa que la conjetura ES trivialmente cierta. Tal vez obviamente eso no es lo que significa para los geómetras, pero creo que otros no geómetras como yo podrían cometer el mismo error. Creo que sería útil mencionar lo que realmente significa un "mosaico".
¿Por qué piensas eso? Eso no es lo que significa en general, donde hay todo tipo de mosaicos que no son cuadrículas cuadradas. Mosaico en matemáticas solo significa que cubres el plano (o lo que sea) con los mosaicos para que no se superpongan, excepto en sus límites.
@Jim, por ejemplo R 3 podrían estar cubiertos por planos en un patrón regular, rotados uno contra el otro. Ni siquiera puedo imaginar cómo funcionaría esto en R 8 .
@gnasher729: Todo lo que quise decir fue que el lenguaje utilizado en la declaración era confuso, pero MJD ahora editó su respuesta para que sea más clara.
¿Qué pasa con los "bordes" de dimensiones superiores? (por ejemplo, para norte = 8 , no hay compartidos ( norte 1 ) -cubos, pero ¿qué pasa con ( norte 2 ) -cubos y así sucesivamente?) ¿O el fracaso de la conjetura para norte > 7 implica que los bordes tampoco necesitan ser compartidos?
@gnasher729 Necesito empezar a vivir en planos de realidad superiores.
Este tema podría hacer una buena contribución a cómo falla la intuición para dimensiones más altas. también.
  • Si tu es un subconjunto abierto de R norte que es homeomorfo a R norte , uno podría pensar que es "obvio" que de hecho es difeomorfo a R norte (quizás pensando algo como "topológicamente parece R norte , y diferenciablemente es localmente trivial"). De hecho, esto es cierto (¡pero de ninguna manera obvio!) para norte 4 . Pero para norte = 4 es falso: existen exóticos R 4 's (variedades diferenciables que son homeomorfas, pero no difeomorfas, para R 4 ), incluidos los "pequeños" que son difeomorfos a un subconjunto abierto de R 4 .

  • Mucho menos profundo, pero divertido: es "obvio" que la suma de dos conjuntos abiertos convexos en el plano cuyo borde es C también tiene un C borde (tal vez pensando en algo como "el borde de la suma está parametrizado por una función suave de los bordes de los sumandos"). Pero esto es falso: de hecho, el borde de la suma siempre es C 20 / 3 (es decir, seis veces diferenciable y con una sexta derivada que es apropiadamente Hölder) y no más en general. Un simple contraejemplo lo dan los epígrafes de X 4 / 4 y X 6 / 6 . Para obtener detalles, consulte Kiselman, "Suavidad de sumas vectoriales de conjuntos planos convexos", Matemáticas. Escanear. 60 (1987), 239–252.

20 / 3 ? ¿¡Qué clase de magia es esa!? +1 ejemplo interesante.

i i es imaginario.            

Recuerdo que me pareció increíble la primera vez que hice este cálculo.
creyendo i i está bien definido es ilusorio.
Ciertamente no estoy diciendo nada profundo. Uno simplemente no debería escribir cosas que parecen expresiones constantes pero no lo son.
i i 0.207879
@bof: Bueno, para ser "obviamente cierto" para mí, es evidente que tiene sentido. De todos modos, no lo consideraría más "obviamente cierto" que " i × i es imaginario".
Ninguno de los infinitos valores para i i son imaginarios :)
@bof: Sí. Y decir que " i i es imaginario" es falso incluso cuando se tiene en cuenta la naturaleza multivaluada . Eso es lo que quise decir.
@xfix Um, no, no lo es.
@anorton Depende de cómo definas "imaginario". La definición habitual es "tener parte real 0". y 0 ciertamente tiene parte real 0.
@Snowbody Ah. Supongo que he visto esa definición bastante a menudo. Normalmente pienso en "imaginario" como en el conjunto C R (De ahí mi confusión)
@Snowbody La definición habitual de "imaginario" es "tener una parte imaginaria distinta de cero". La definición de "puramente imaginario" es "tener parte cero real".
@AnónimoPi i = Exp ( ( 4 norte + 1 ) π i 2 ) dónde norte norte . Por eso i i = Exp ( ( 4 norte + 1 ) π 2 ) .
@bof Pensé que "tener una parte imaginaria distinta de cero" era la definición de no real. Algunos números son reales, algunos números no lo son. Algunos números son imaginarios, algunos números no lo son. Los números reales, los números imaginarios y otros, son todos complejos. Wikipedia está de acuerdo conmigo, por todo lo que vale... (no mucho)
Los números imaginarios puros y los números reales forman una descomposición de suma de dirección de los números complejos. Para que esto sea natural, el cero debe ser tanto real como imaginario puro.
¿Es esto realmente intuitivamente imaginario? ¿No se podría intuir que es complejo con parte real distinta de cero y, por lo tanto, no imaginario? ¿O podría no haber intuición y, por lo tanto, solo se necesita hacer un cálculo simple?
@ Dr.MV ¿Cómo puede ser negativo?
@AkivaWeinberger Tenemos
i i mi i registro ( i ) = mi i × i ( π / 2 + 2 k π ) = mi π / 2 2 k π
Entonces, i i ( 0 , )
... y i i es racional

Esta parte es verdadera (teorema de separación de Jordan-Brouwer):

(a) Cualquier incrustación del 2 -esfera en 3 El espacio euclidiano bidimensional separa el espacio en dos regiones disjuntas.

Pero esta parte, que parecería ser una generalización natural del teorema de la curva de Jordan-Schönflies, no es cierta:

(b) Las regiones son homeomorfas al interior y al exterior de la esfera unitaria.

Yo mismo estaba pensando en este ejemplo (la esfera con cuernos). Mi respuesta se puede ver como una especie de análogo 2d de alguna manera ...
Este es el primer resultado en este hilo que realmente encuentro poco intuitivo. ¿Que demonios?

Realmente me gustan las "pruebas incorrectas", ya que normalmente la idea de por qué la prueba es incorrecta te da una idea del tema. Una versión muy simple es esta, que lancé en mis primeros semestres cuando era tutor:

Cada relación binaria que es simétrica y transitiva es también reflexiva y por lo tanto una relación de equivalencia.

"Prueba":

Dejar denote una relación simétrica y transitiva y sea X , y ser dos elementos con X y . Como es simétrica, se cumple que y X . Desde X y y y X se sigue por la transitividad de eso X X , que es la definición de reflexividad.

Editar: como me preguntaron, he aquí por qué la prueba es incorrecta (mueva el mouse allí para mostrar):

Eche un vistazo a la relación vacía en un conjunto no vacío S , para que no haya X , y S de modo que X y . Esta relación es simétrica y transitiva, pero no reflexiva. necesidades de reflexividad X X mantener para todos X . La prueba supone que hay ay, por lo que x ~ y, que no es necesariamente el caso para todos X .

¿Dónde está la falla en este argumento?
@ user21820 Lo siento, por formular demasiado ambiguo. Edité y debería quedar claro ahora lo que significa.
Lo siento, leí mal.
Una relación no es simplemente "reflexiva"; más bien es reflexivo en algún conjunto. La relación en { 1 , 2 , 3 , 4 } por donde 3 3 , 4 4 , 3 4 , y 4 3 , y nada más está relacionado, no es reflexivo en el conjunto { 1 , 2 , 3 , 4 } , pero sí hay un conjunto sobre el que es reflexivo.
Esta situación se trata en detalle en la página 30 del libro ESSENTIALS OF ABSTRACT ALGEBRA de Bundrick y Leeson (1972).
Las relaciones transitivas simétricas comprenden una clase muy importante de relaciones, a veces llamadas relaciones de equivalencia parcial . Así como puede modificar una relación de equivalencia para obtener un cociente, puede restringir y modificar una relación de equivalencia parcial para obtener un subcociente, que es un subconjunto de un cociente (o equivalentemente un cociente de un subconjunto). …

Este es uno de mis favoritos: supongamos que jugamos con una moneda justa.

Teorema (falso) En un juego largo de tirar una moneda, cada jugador estará en el bando ganador durante aproximadamente la mitad del tiempo, y la ventaja pasará no pocas veces de un jugador a otro.

Lo siguiente es del clásico de W. Fellers de Introducción a la teoría de la probabilidad y sus aplicaciones , Vol 1:

De acuerdo con creencias generalizadas, la llamada ley de los promedios debería garantizar el teorema anterior. Pero, de hecho, este teorema es incorrecto y contrario a la creencia habitual , se cumple lo siguiente:

con probabilidad 1 2 no se produjo ningún empate en la segunda mitad del juego, independientemente de la duración del juego. Además, las probabilidades cerca del punto final son mayores .

De hecho, esto conduce a la ley del arco seno para las últimas visitas (ver, por ejemplo, Vol. 1, cap. 3, sección 4, Teorema 1).

Nota: tenga en cuenta las declaraciones notables citadas del Capítulo III: Fluctuaciones en el lanzamiento de monedas y paseos aleatorios :

Por ejemplo, en varias aplicaciones se supone que las observaciones de un juego individual de lanzamiento de monedas durante un largo intervalo de tiempo producirán las mismas características estadísticas que la observación de los resultados de un gran número de juegos independientes en un instante dado. Esto no es así.

y más adelante:

De todos modos, es lógico pensar que si incluso el simple juego de lanzar una moneda conduce a resultados paradójicos que contradicen nuestra intuición, esta última no puede servir como guía confiable en situaciones más complicadas.

[2015-07-16] Según un comentario de @HenningMakholm, algunos ejemplos exponen aspectos llamativos.

  • Suponga que una gran cantidad de juegos de lanzamiento de monedas se llevan a cabo simultáneamente a razón de uno por segundo, día y noche, durante todo un año. En promedio, en uno de cada diez juegos el último empate ocurrirá antes 9 han pasado días , y el cliente potencial no cambiará durante los siguientes 356 días. En uno de cada veinte casos, la última igualación tiene lugar dentro de 2 1 2 días, y en uno de cada cien casos ocurre dentro de los primeros 2 horas y 10 minutos.

  • Supongamos que en un experimento de aprendizaje que dura un año, un niño se retrasa constantemente, excepto, quizás, durante la semana inicial. Otro niño estaba constantemente por delante excepto, quizás, durante la última semana. ¿Los dos niños serían juzgados iguales? Sin embargo, deja que un grupo de 11 los niños estén expuestos a un experimento de aprendizaje similar que no involucre inteligencia sino solo azar. uno entre los 11 aparecería como líder durante todas menos una semana, otro como rezagado durante todas menos una semana.

Los ejemplos anteriores son, de hecho, una consecuencia de la ley Arcseno para las últimas visitas .

¿Podría agregar alguna explicación a la tercera cita sobre cómo puede coexistir con la suposición abreviada habitual de que un "lanzamiento de una moneda" es independiente de cualquier otro lanzamiento de moneda en cualquier lugar del espacio-tiempo? ¿Cómo "sabrían" las monedas observadas en el juego largo que se están utilizando en un juego largo en lugar de llevarse a cabo simultáneamente en diferentes juegos?
@HenningMakholm: Agregaré información. De hecho, entre bambalinas se reduce a un aspecto puramente combinatorio. Hay más posibilidades de conservar o aumentar la altura dentro k escalones que bajar. Tal vez podríamos decir informalmente que no es la moneda, que sabe algo sobre su camino, sino el espacio circundante con la estructura de celosía específica, que tiene un cierto conocimiento de la moneda incrustada y su altura actual. :-)
Probablemente la información más necesaria es cuáles son las "observaciones" de las que hablan. Tal como se cita, parece que afirman que puede entregarles una lista de 1000 lanzamientos de monedas que se originan en un juego largo o en 1000 juegos diferentes, y podrían adivinar cuál con más de 1/2 probabilidad, por algún tipo de análisis estadístico.
@HenningMakholm: He agregado dos ejemplos que podrían ser útiles. Atentamente,
Tenga en cuenta que "las observaciones de un juego individual de lanzamiento de monedas durante un largo intervalo de tiempo producirán las mismas características estadísticas que la observación de los resultados de una gran cantidad de juegos independientes en un instante dado". en realidad puede ser (algo) cierto, si lo hace de la manera correcta (es decir, mirando escalas de tiempo exponenciales). Proviene de la convergencia de la trayectoria a un movimiento browniano, la invariancia de escala del movimiento browniano y la ergodicidad/mezcla del flujo de cambio de escala del movimiento browniano.
D.Thomine: Información interesante. ¡Gracias!
¿Puede proporcionar una referencia al libro " Chung & Feller Introducción a la teoría de la probabilidad y sus aplicaciones, Vol. 1 " que sugiere? Conozco el libro clásico del mismo nombre pero es solo de Feller y otro es Elementary Probability Theory de Chung y AitSahlia.
@ApoorvPotnis: Lo siento, esto fue un error. También me refiero al libro de W. Fellers. La ley del arco seno fue probada por primera vez por KL Chung y W. Feller y parece que la confundí. He actualizado la publicación. Atentamente,

Teoremas que son intuitivamente verdaderos, pero en realidad defectuosos:

  • No existe una función real continua, diferenciable en ninguna parte.

  • No existe una función real que sea derivable y no monótona en ningún intervalo no trivial.

  • Si una función real satisface X , y , F ( X + y ) = F ( X ) + F ( y ) , es de la forma X a X .

  • Las sumas infinitas y las integrales se pueden intercambiar en cualquier momento.

  • Un espacio métrico conexo es conexo por caminos.

¿Por qué es intuitivamente cierto que las sumas e integrales infinitas se pueden intercambiar en cualquier momento? Creo que es relativamente simple encontrar un contraejemplo.
¿Puedes mostrar un contraejemplo para la tercera afirmación? (El de las funciones lineales)
Casi cualquier afirmación en la que el contraejemplo comience con "Vamos H Sea una base Hamel de R …” va a ser una de esas cosas que parecen ciertas pero no lo son.
Siento que el de una función lineal realmente no encaja. ¿No está definida la "función lineal" por esa propiedad más F ( a X ) = a F ( X ) ? ¿No sería solo no obvio para alguien que malinterpreta el uso de "función lineal"?
@ jpmc26 no es la definición F ( a X + b y ) = a F ( X ) + b F ( y ) o F ( a X ) = a F ( X ) y F ( X + y ) = F ( X ) + F ( y ) ?
"No existe una función real que sea diferenciable y no monótona en ningún intervalo no trivial". ¿En serio? ¿Cuál es el contraejemplo?
@Alessandro Dije, "esa propiedad más ..." para referirme a la de la respuesta.
Para el #4, la restricción donde este _sí funciona está dada por el teorema de Fubini-Tonelli

La siguiente declaración que una vez creí que era "obvia":

Si F : R [ 0 , ) continua es tal que F ( X ) d X < , entonces límite X ± F ( X ) = 0

que en realidad es falso.

(Nota: es cierto si F es uniformemente continuo!)

¿Puedes dar un contraejemplo?
Esto es bastante fácil de dar un contraejemplo a: dejar F tener protuberancias tan abajo como quieras, solo con anchos lo suficientemente pequeños para que la suma de sus integrales sea cero. Puedes hacer Lim sup X F ( X ) = Por aquí.
@hallaplay835 cf. Libro Real Analysis de Stein&Shakarchi, el capítulo sobre L 1 funciones Fue un ejercicio.
@hallaplay835 Si F ( X ) = porque ( X 2 ) , entonces claramente F ni siquiera tiene un límite como X se acerca al infinito. Pero la integral de F existe como una integral de Riemann impropia (Sugerencia: haga un cambio de variable y = X 2 y d X = 1 2 ( y ) ¡y la prueba de Abel hace el resto!).
puedes asumir F ser positivo. Eso hace que la condición sea un poco más engañosa. De hecho, esto es exactamente lo que se afirma en muchos libros de mecánica cuántica en un intento de probar el decaimiento de la función de distribución en el infinito.

Hay un buen número de situaciones de probabilidad contrarias a la intuición. Uno de mis favoritos es dados no transitivos:

Hay 3 dados, A, B y C. Los dados tienen números del 1 al 9 en sus lados (repeticiones posibles). Si el B vence (número más alto) al A más de la mitad de las veces y el C le gana al B más de la mitad de las veces, entonces el C vencerá al A más de la mitad de las veces.

Esta no es necesariamente una afirmación verdadera. Los dados se pueden diseñar de tal manera que la propiedad "x vence a y" no sea transitiva. A vence a B, que vence a C, que vence a A.

Hay una serie de declaraciones contrarias a la intuición similares en la votación de múltiples vías, de las cuales esto podría verse como un caso especial. Por ejemplo, en las competiciones de patinaje artístico a nivel de campeonato hay nueve jueces. A veces sucedía que la mayoría de los jueces clasificaban al patinador A encima del patinador B , la mayoría clasificaría B arriba C , y la mayoría clasificaría C arriba A . Las reglas de puntuación tenían que tener condiciones para hacer frente a este tipo de situación.
De hecho, puede crear cualquier gráfico dirigido loco entre los candidatos que desee, y hay una población de votantes de orden lineal aparentemente racional cuyo comportamiento de votación dará como resultado ese resultado. Resulta que el ÚNICO comportamiento de los votantes que resulta en decisiones mayoritarias consistentes es seguir ciegamente la línea del partido.
¿Es nueve el número mínimo de lados/jueces necesarios para lograr el fenómeno de no transitividad?
Definitivamente no. La mayoría de los dados no transitivos se describen como de 6 caras. Es solo que los números en ellos son del 1 al 9 (no necesariamente todos usados).
@Duncan, ¿cuál es el número mínimo de lados para los dados? ¿podría citar un ejemplo de esa votación?
El número mínimo de lados sería 3 (aunque es difícil tener un dado de 3 lados). Pero toma estos como ejemplos de todos modos: dado A: 2,4,9 dado B: 1,6,8 dado C: 3,5,7 dado A vence al B 5/9 del tiempo, dado B vence al C 5/ 9 de las veces, el C le gana al A 5/9 de las veces. (Este es solo el ejemplo de la página de wikipedia en dados no transitivos pero con el número de lados cortados por la mitad).
@duncan Suponga que tiene tres patinadores, XY y Z. El juez 1 prefiere X a Y e Y a Z; el juez 2 prefiere Y a Z y Z a X; el juez 3 prefiere Z a X y X a Y. Ahora dos de los tres jueces prefieren X a Y; dos prefieren Y a Z, y dos prefieren Z a X.

Los números reales/conjunto de Cantor son contables.

Hay varias pruebas falsas "obvias":

  1. "Prueba" . Considere el árbol { 0 , , 9 } norte , entonces cada número real corresponde a un nodo en el árbol. Dado que solo hay muchos niveles contables y cada uno es finito, se deduce que los números reales son finitos.

    ¿Por qué falla? Este conjunto en realidad no es un árbol. Puede ordenarlo para que parezca un árbol, pero de hecho el árbol estaría compuesto de segmentos iniciales de cada función ordenados por continuación. Este árbol, entonces, tendría un último nivel (a saber, un nivel que en ningún punto tiene un sucesor), y sería exactamente el nivel de las funciones mismas (los niveles anteriores serían segmentos iniciales propios de las funciones).

    Si eliminamos ese último nivel, entonces el árbol sí es contable, pero ahora cada número real corresponde a una rama del árbol en lugar de a un nodo. (Es la rama única cuyo límite es igual a la función, que apareció previamente en ese nivel final).

  2. "Prueba" . Los números racionales son contables, y entre cada dos números reales hay un número racional. Por tanto esto define una biyección entre pares de números reales y los números racionales.

    ¿Por qué falla? Debido a que hay muchos, muchos, muchos pares asignados al mismo número racional, esto no es realmente una biyección.

  3. "Prueba" . El conjunto de Cantor es cerrado, su complemento es abierto, por lo que es una unión contable de intervalos, por lo que el conjunto de Cantor es contable.

    ¿Por qué falla? Porque no todos los puntos del conjunto de Cantor son extremos de dicho intervalo. Por ejemplo 1 4 . Sin embargo, es cierto que los extremos de estos intervalos forman un subconjunto denso contable.

  4. ¡PRIMA! , PAG ( norte ) es contable.

    "Prueba" . Por cada finito norte , PAG ( norte ) es finito, y PAG ( norte ) = PAG ( norte ) = PAG ( norte ) , es una unión contable de conjuntos finitos, que es contable.

    ¿Por qué falla? Debido a que la unión solo incluye subconjuntos finitos de norte , pero ninguno de sus infinitos subconjuntos.

Creo que la inexistencia de números irracionales puede estar en esta categoría (obviamente cierto, pero falso), pero se descubrió que era falso hace tanto tiempo que ya no recordamos lo extraño que debió parecer.
@MJD: Cierto, y en dos siglos les parecerá extraño que en algún momento la gente pensara que R puede ser contable. O al menos eso es lo que espero!
Ahora bien, me parece extraño que nadie antes de Cantor observara que si R fueran contables, podría estar cubierto por una familia de intervalos disjuntos cuya longitud total fuera menor que cualquier ϵ .
@MJD: Hmm, es un poco extraño. ¡Tal vez ejecutaron a todas aquellas personas que sugirieron eso antes de que pudieran publicar su hallazgo! ;-)
¿En qué sentido tiene su árbol un “nivel final” y qué significa ese término aquí?
@Christopher: ni siquiera es un árbol, estrictamente hablando. Pero cuando se presenta esta prueba, a menudo se argumenta que el árbol son los segmentos iniciales de una función de norte a { 0 , , 9 } . El nivel final es la función misma. Y significa que estos nodos no tienen sucesores (y ellos mismos no son sucesores de ningún otro nodo).
Dado que la expansión decimal de, digamos, 1 / 3 , no tiene un último dígito, el “árbol” no tiene un nivel final en este sentido.
@Christopher: No, el árbol tiene ω + 1 niveles La última es la secuencia infinita. 0.333 .
He hecho la paz mental con el hecho de que PAG ( norte ) es incontable hace mucho tiempo. De hecho, se ha arraigado tanto en mi mente que encuentro difícil aceptar que el conjunto de subconjuntos finitos de norte es contable!
@triple_sec: Jaja, sí, eso también puede pasar. Aunque es una de esas cosas que si lo piensas un rato, vuelven a aparecer.
@Asaf Hilarantemente, con tu ω + 1 has usado la idea del concepto erróneo común " 0.999 < 1 porque difieren en una cantidad 0.000 1 " en un argumento correcto sobre otro teorema muy relacionado y muy cuestionado.
@Ryan: No veo cómo. Los objetos en el árbol no son números reales. son funciones de norte a { 0 , , 9 } . Podemos identificarlos como números reales, pero esta identificación no tiene por qué ser uno a uno; solo sobre. Y eso es exactamente lo que está pasando aquí. Además, los elementos de la ω + 1 no es una secuencia de tipo orden ω + 1 , sino más bien una secuencia de tipo de orden ω .
@Asaf: no quise decir que en realidad usaste secuencias decimales de tipo ω + 1 , sino simplemente que construiste números reales, como decimales, a través de un proceso inductivo con ω + 1 pasos, donde cada uno de los primeros ω es un decimal finito al que se aplica correctamente la creencia errónea.
@Ryan: Nuevamente, eso no es lo que hice. No construí ningún número real. Simplemente dije que hay una manera de identificar cada número real como un nodo en el árbol. Algunos pueden tener más de una manera.
@Asaf, no veo cómo obtienes el tipo de orden ω + 1 entradas en elementos del conjunto { 0 , , 9 } norte . Por ejemplo, creo que estamos de acuerdo en que hay exactamente un elemento en este conjunto correspondiente a la expansión decimal de π 3 . Si ese elemento, como dices, tiene tipo de orden ω + 1 , entonces tiene un último dígito. ¿Cuál es ese y por qué? ¿Y cómo llega a estar en una función de norte a { 0 , , 9 } ?
@Christopher: La altura del árbol es ω + 1 . No ω . Para ser explícito, permítanme definir exactamente qué es el árbol, el norte -ésimo nivel (donde norte < o metro mi gramo a + 1 ) es exactamente el conjunto de F norte . Y s < t si s t (lo que significa que ambos pueden verse como la restricción de la misma función). El elemento no tiene "tipo de orden" y creo que no entiendes algo bastante fundamental aquí, pero no estoy seguro de qué es exactamente.
@Asaf, tal vez podría aclarar qué quiere decir exactamente con "altura" aquí. Cada elemento del conjunto que definiste claramente tiene un orden. ω , y aunque creo que queremos decir lo mismo para "el norte -el nivel” cuando norte es finito, si leo F norte como F limitado al conjunto de preimagen { 0 , , norte } . Ahora, ninguno de estos está realmente en su conjunto. Cada elemento de su conjunto es la unión de un número contable de tales secuencias finitas, que puede elegir ver como caminos finitos. Pero, ¿qué sería, con su definición de "altura", un árbol de tal altura ω ?
@Christopher: Sí, eso es exactamente lo que está escrito en mi respuesta, si quiere leerlo. El conjunto { 0 , , 9 } norte NO es un árbol, pero podemos pensar en él como un conjunto que se comporta un poco como un árbol en algún sentido. Pero el árbol real es la restricción de cada función a un segmento inicial del dominio. Hay ω segmentos iniciales adecuados , y luego hay un segmento inicial más que es la función real. Esto define un árbol que tiene altura ω + 1 . Y por la altura del árbol me refiero al tipo de orden de sus niveles, ya que hay ω + 1 niveles, esa es la altura.
@MJD ¿No fue la observación de que R ¿No es contable necesario para la observación de Cantor?
@Brilliand: Esa fue la observación de Cantor.
Toda esta confusión sobre la no prueba 1. ¿No podemos simplemente preguntar a qué número natural se supone que corresponde π ?
@Cory: O 1 3 , Sí. Hay muchos ejemplos. Pero esto no explica por qué falla la demostración. Simplemente demuestra que lo hace. Estaba tratando de explicar realmente la razón del fracaso.

Hipótesis: Toda función infinitamente diferenciable es real-analítica en alguna parte.

Esto es falso, como lo muestra (por ejemplo) la función de Fabius .

Vea esta interesante respuesta de Dave L Renfro y las publicaciones a las que se vincula.

Me sorprende que nadie haya dado esta respuesta ya, así que aquí está:

Hay más números enteros que números naturales.

Es obvio, ¿no?

Dudo con este, porque depende de la definición (algo poco sólida) de "más".
Absolutamente hay más números enteros que números naturales (por inclusión). Y la cardinalidad es una noción un poco artificial si lo piensas por un momento.
@Tibor: No, la cardinalidad no es una noción artificial si lo piensas por un momento .
Todas las matemáticas son artificiales, por definición. :)
“Me sorprende que nadie haya dado esta respuesta ya”. En realidad, el cuarto párrafo de esta respuesta , "Hay tantos números pares como números naturales", se publicó el 5 de junio a las 13:58 (implícitamente, enumera cosas que son verdaderas pero no obvias), y este comentario , "... hay la mitad de enteros impares" [como enteros], se publicó el 6 de junio a las 7:30.
Estos son esencialmente equivalentes a su publicación: la afirmación de que, si existe una correspondencia uno a uno entre un conjunto (no vacío) y un subconjunto estricto (en particular, "la mitad de") otro conjunto (no vacío), entonces no puede haber una correspondencia biunívoca entre los conjuntos en su totalidad. Como mencioné en otro comentario, creo que esto es cierto si los conjuntos son finitos, que es lo que lo hace obvio.
Estoy de acuerdo con GOTO o porque no solo el hecho de que los enteros son básicamente un mejor nombre para los números enteros y los números naturales son solo enteros/enteros positivos, sino también porque puedes calcular cuántos enteros se comparan con los números naturales si dejamos que sea el número de enteros y n sea el número de números naturales y obtenemos... i = 2n + 1 y n = (i - 1)/2... porque hay tantos números naturales como enteros negativos (que es básicamente el opuesto y debe haber el mismo número de opuestos que regulares) + 0 que son dos n más 0 que cuenta como un número adicional.
Primero, era extraño que hubiera tantos números naturales como enteros. Pero a lo largo de los siglos, la gente se acostumbró a eso, hasta que se sorprendió de que realmente hay más números reales que números racionales.

¡La imagen de una medida cero establecida bajo un mapa continuo tiene una medida cero!

¿No es verdad?!?!
@LeilaHatami No, mira aquí . Esencialmente, podemos mapear biyectivamente (y continuamente) el conjunto de cantor a [0,1], lo que mapea un conjunto de medidas 0 a un conjunto de medidas 1.

La probabilidad de que le des a cualquier punto de un tablero de dardos es 0 pero la probabilidad de que le des al tablero de dardos es 1 (siempre y cuando no seas tan malo como yo lanzando dardos ;D).

EDITAR:

Como señaló @JpM, no seguí el formato de estas publicaciones, aunque la idea puede entenderse ( fácilmente en mi opinión ) a partir de lo que he dicho anteriormente.

Pseudoafirmación: la probabilidad de acertar en un solo punto en un tablero de dardos es mayor que 0 ya que la probabilidad de acertarle (asumiendo que acertarás en el tablero de dardos) es 1 .

Parece obvio en el sentido de que un montón de 0 no puede sumar para ser 1 por lo que cada punto debe tener alguna probabilidad. En realidad falso debido a algunas propiedades de las medidas.

... ¿no es este un teorema no obvio que es realmente cierto?
@JpMcCarthy los laicos podrían afirmar que "la probabilidad de que lleguemos a un solo punto es mayor que 0 "entonces de hecho este es un teorema que en realidad es falso.
En particular, hay una intuición "obvia" de que el punto en el que realmente golpeaste debe haber tenido una probabilidad mayor que 0 , "ya que la probabilidad 0 los eventos son imposibles". Supongo que este ejemplo es tan obvio que tiene que hacerse axiomáticamente falso en la teoría para sacarlo del camino y hacer algún trabajo :-)
¿Qué sucede si afirmo que la probabilidad de acertar es un número hiperreal no arquimediano?
@DanielV Diría una de dos cosas según mi estado de ánimo: a) Una persona común no diría tal cosa, es decir, no diría que la probabilidad de alcanzar un solo punto es infinitesimal. b) Generalmente (de hecho no he visto otra cosa, entonces de nuevo soy joven e ingenuo) las medidas se definen con valores en los números reales extendidos, es decir sin infinitesimales.
Estoy confundido. ¿Quizás quiere decir "La probabilidad de que golpee cualquier punto en un tablero de dardos no es cero"? Eso sería obvio y falso.
@JpMcCarthy eso es exactamente lo que dije en el comentario.
...entonces esa debería ser tu respuesta! Tu respuesta es un teorema no obvio que en realidad es verdadero en lugar de un teorema obvio que en realidad es falso.
@JpMcCarthy ok, cambié el formato de mi publicación para seguir exactamente cómo se hizo la pregunta.
@JpMcCarthy Gracias; me gustan más los de arriba, pero este fue interesante.
¿Debería suponerse que "punto" significa "región en el tablero de dardos tal que el área es 0" (o quizás "región de dimensión 0 en el tablero de dardos", aunque la propiedad también sería válida para una línea unidimensional) ), que no necesariamente coincidiría con la definición del profano de "punto"?
@JAB Creo que estás complicando demasiado esto, un laico no va a pensar esto en profundidad sobre la definición de "punto". Estoy considerando que un punto es exactamente lo que pensaría un laico, un lugar exacto en el tablero de dardos.
Creo que lo esencial que algunas personas simplemente no pueden entender es que un evento con probabilidad 0 aún puede suceder.
@Neil, sí, eso es exactamente a lo que estaba tratando de llegar aquí; D
Esta no es una afirmación matemática, ni es falsa.
¿No es esto análogo a que una integral de a a a de una f integrable para 0 < a < 1 debe ser > 0 si la integral de f de 0 a 1 es distinta de cero?
@scibuff, de hecho, es equivalente, dada la configuración correcta

Mi "teorema":

La declaración Todos aman a mi bebé, pero mi bebé no ama a nadie más que a mí, se trata de un par de amantes.

Es tan simple y tan obvio que hasta mi abuela lo entenderá. Y por más que expliques el cálculo lógico simple que muestra que aquí estamos hablando de un solo narcisista, la mitad de la clase de estudiantes de lógica de primer semestre seguirán insistiendo en que tu prueba está mal, y no saben qué está mal. al respecto, pero no puede referirse a una sola persona.

Algunas personas tratan el amor como una relación que no se define reflexivamente.
@Joshua Todavía tengo que conocer a un estudiante que se da cuenta de que esta es la raíz del problema y explicar cómo evitarlo en la "prueba" lógica. Tienes razón, por supuesto, una vez que definimos el conjunto de "todos" no como "todos los humanos" sino como "todos los humanos que son capaces de amar a mi bebé", y decidimos que mi bebé no está en el conjunto, el resultado es diferente. .
No tengo idea de lo que esta respuesta está tratando de decir.
@goblin Como se puede ver en esta prueba en lógica, el único significado posible de la declaración es que mi bebé soy yo .

Considere una función F : ( 0 , ) R eso es C en ese intervalo. A primera vista, se podría pensar que, si límite ( F ) = 0 como X , entonces límite ( F ) = 0 como X . Sin embargo, esto es falso. Aquí hay solo un contraejemplo:

F ( X ) = 1 X pecado ( X 2 )

Además, si añadimos la estipulación de que F aunque sea monótona, todavía se pueden encontrar contraejemplos (aunque bastante patológicos).

Tu "otro" ya fue mencionado.........
Vaya, no lo había visto allí. Gracias.
¿Podrías darme un C monotónicos, contraejemplos? ¡Muchas gracias!
Sí, puede obtener uno jugando un poco con las respuestas aquí: math.stackexchange.com/questions/788813/…

Un arco simple (imagen homeomorfa del intervalo unitario cerrado) en el plano tiene 2 Medida de Lebesgue -dimensional cero.

esto es obvio, ¿verdad?
@athos Debería ser obvio, de lo contrario no sería una respuesta válida a esta pregunta. ¿Estás preguntando por qué es realmente falso? Probablemente haya una construcción directa más simple de un "arco grueso" en el plano, pero se sigue de un teorema más general de RL Moore y JR Kline ( En el plano más general, conjunto cerrado de puntos a través del cual es posible pasar un simple arco continuo , Ann. of Math.(2) 20 (1919), 218-223) que cada conjunto de Cantor (homeomorfo del estándar) en el plano está contenido en un arco; aplique esto a un conjunto de Cantor gordo en el plano.
gracias por el enlace... un poco difícil para mí, lo marcaré para una digestión posterior :)

Cualquier número real se puede calcular de alguna manera.

Más formalmente:

Para cada número real, existe un programa de longitud finita que calcula ese número.

Dado que los números reales son incontables, mientras que los números computables son contables, ese no puede ser el caso.

Esta limitación proviene del hecho de que estamos atascados en el uso de programas de longitud finita. Los programas de longitud infinita se pueden definir para calcular cualquier número real (trivialmente). Entonces, hay un sentido en el que todos los números reales se pueden calcular.

Simplemente no por los humanos. Tenga en cuenta que, dado que un solo programa de longitud infinita ocuparía una memoria infinita (y no parece que tengamos computadoras/cerebros infinitos), la mayoría de estos programas de longitud infinita nunca se pueden conocer, y mucho menos calcular. Entonces, los números computables son solo aquellos números computables por un programa de longitud finita. Y el conjunto de programas de longitud finita es contable.

Lo que personalmente creo que debería ser es... Para cada número real no transdental existe un programa de longitud finita que calcula ese número...
@BinaryFreak Creo que los no trascendentales son todos algebraicos y, por lo tanto, computables, por lo que sería un teorema obvio que resulta ser cierto. En una nota relacionada interesante, mientras que cada número no computable es trascendental, algunos trascendentales son computables ( π , mi , y algunos reales no repetidos como 0.123456789101112... ).

Teorema: Sea F 1 ( X , y ) y F 2 ( X , y ) ser dos densidades de probabilidad conjuntas, cada una con su X , y componentes correlacionados positivamente ( C o v 1 ( X , y ) > 0 , C o v 2 ( X , y ) > 0 ). Dejar F 3 = α F 1 + ( 1 α ) F 2 Sea la densidad de mezcla, para algunos 0 α 1 . Entonces C o v 3 ( X , y ) > 0 .

En palabras: la mezcla de poblaciones conserva el signo de correlación. En otras palabras: si el usuario masculino promedio de MSE es más brillante que la media, y si la usuaria femenina promedio de MSE es más brillante que la media, entonces el usuario promedio de MSE es más brillante que la media. Obviamente cierto.

FALSO. Véase la paradoja de Simpson .

Quizás el siguiente ejemplo hará que la obviedad sea más evidente. Equipo de beisbol A tiene una mejor proporción de victorias y derrotas que el equipo B en la primera parte de la temporada. Luego hay una huelga y se pierden algunos juegos. Cuando se reanude la temporada, el equipo A también tiene una mejor proporción de victorias y derrotas que el equipo B en la segunda parte de la temporada. Por lo tanto, equipo A tiene una mejor proporción de victorias y derrotas que el equipo B en general. (¡Incorrecto!) El béisbol jugó una temporada tan dividida en 1981, pero no sé si la paradoja realmente ocurrió entonces. Probablemente no, ya que requiere A y B para jugar números muy diferentes de juegos.

Afirmar:

Si el producto escalar de dos vectores es 0, entonces son linealmente independientes.

Mi profesor me lanzó esta pregunta hoy y me enamoré.

jaja, me tomó un segundo.
depende de que campo. Muy bonita, pero una pregunta capciosa. Me siento engañado.
@Lost1: en realidad no depende del campo: v 0 = 0 y { v , 0 } nunca es independiente.
@JasonDeVito ah... estaba pensando tomar Z 2 2 y el vector ( 1 , 1 ) ...
@ Lost1 De acuerdo con la definición habitual, el producto punto debe tener un valor real (ciertamente no un valor de campo finito; no son campos ordenados, y sin orden, la definición positiva no tiene sentido. Y sin eso, este teorema es falso pero tampoco tiene razón para ser verdad!)
@RyanReich No necesariamente. Es bastante estándar en álgebra lineal (y teoría de números) considerar productos internos que toman valores en el campo subyacente de los espacios vectoriales en consideración. Solo cuando se especializa en campos de valores reales o complejos, la definición positiva entra en escena. Desde ahora es posible que tengamos v v = 0 para v 0 , se utiliza la noción de vectores isotrópicos .

En mi opinión, los resultados más interesantes (pero también a veces poco intuitivos) en matemáticas son aquellos que enuncian un teorema que acaba siendo falso porque en realidad se cumple en muchos casos, salvo en muy pocos casos o muy extraños. En otras palabras, los teoremas falsos más "obvios" para mí son aquellos que tienen contraejemplos muy difíciles.

Algunos ejemplos:

  • Banach-Tarski: Existe un subconjunto estricto A de la euclidiana norte -pelota B tal que uno puede particionar A y B en un número igual de subconjuntos adicionales que pueden mapearse entre sí mediante isometrías. Esto demuestra que no todos los conjuntos son medibles y que es posible realizar particiones que no conservan la medida.

  • No finitud de estructuras diferenciables: Por R norte con norte = 4 , hay un número incontable de estructuras diferenciables distintas.

  • Divergencia de la serie de Fourier: Existe una función integrable en [ π , π ] cuya serie de Fourier diverge en todas partes. Esto es extremadamente inusual porque para cualquier función típica que podamos escribir, por lo general, su serie de Fourier puede divergir en uno o en un número finito de puntos, pero probablemente convergerá en cualquier otro lugar.

Esto me hace pensar en: un subconjunto de R norte está cerrado o abierto (pero no ambos). Lo cual, por supuesto, no es cierto porque el conjunto vacío es abierto y cerrado. (Pero también, no creo q está cerrado o abierto en R ).
No necesitas un ejemplo extraño para encontrar un subconjunto de R norte que no es ni abierto ni cerrado; simplemente tome una bola cerrada y elimine un punto límite; o tome una secuencia de puntos que converge a un límite, pero omita el límite. La terminología puede llevar a pensar que los conjuntos son abiertos o cerrados, pero no hay nada en los conceptos reales que lo haga parecer así.
¿Cuál es un ejemplo de función con series de Fourier divergentes en todas partes?
@Ruslan, el ejemplo no es trivial y se debe a Kolmogorov. Una construcción divergente anterior, casi en todas partes, fue uno de los primeros reclamos de fama de Kolmogorov. Tendrás que consultar a Google sobre esto, no conozco el ejemplo.
@Ruslan Google nos lleva de regreso aquí .

Un teorema 'obvio' pero falso: hay más conjuntos abiertos en R 2 (o R norte ) que hay números reales.

Y en una línea similar tenemos este corolario de la primera afirmación: Hay más funciones continuas R R que hay números reales.

(Ambas afirmaciones son falsas).

Estas son algunas de las declaraciones falsas que me vinieron a la mente y que me hicieron levantar al menos una ceja cuando me di cuenta por primera vez de que no eran ciertas.


Toda función lineal entre dos espacios vectoriales es continua.

Cierto solo mientras el dominio sea de dimensión finita. Si no lo es, entonces existe una función lineal que no es continua, ¡en ningún punto!


El conjunto de números reales de ninguna manera puede estar (totalmente) ordenado de tal manera que cada conjunto no vacío tenga un elemento mínimo.

Falso si se supone elección, por el teorema del buen orden.


q no es contable.

Todavía estoy tentado a creerlo a veces...


Si la derivada de una función real a real continua existe en casi todas partes y (dondequiera que exista) se desvanece en casi todas partes, entonces la función debe ser constante.

FALSO. De hecho, existe una función que satisface la premisa y es estrictamente [ sic! ] aumentando!


Todo conjunto compacto es cerrado.

El nombre “compacto” sugeriría esto, pero esto solo se puede garantizar en los espacios de Hausdorff.


Un conjunto es compacto si y solo si cada secuencia contiene una subsecuencia convergente.

Si bien es cierto en espacios métricos, no solo es falso en algunos espacios topológicos más generales, sino que ¡ninguna condición implica la otra!

Creo totalmente que q es contable. Querías decir R ? Para lo siguiente, ¿supone que la derivada existe en todas partes, o solo en casi todas partes?
no, quise decir q . Quiero decir, duh, q es contable, pero siempre necesito un acto de fe para creerlo. Si puedo enumerar los racionales en [ 0 , 1 ] , ¿por qué no puedo hacerlo en orden creciente? (Es decir, tal que q 1 < q 2 < , dónde { q norte } norte norte = [ 0 , 1 ] q ). En cuanto a la otra “paradoja”, quise decir que “existe y desaparece” es una intersección de eventos cuyo complemento tiene medida cero. La diferenciabilidad no se asume en todas partes. Lo dejaré más claro, gracias por hacerme darme cuenta de la ambigüedad.
me he convencido de que Z 2 es contable; obviamente es posible idear un algoritmo para contarlos. Y me he convencido de que esto significa que q es contable porque q son pares ordenados de elementos en Z 2 , por lo que hay "menos" elementos en q que Z 2 (pensando en elementos gráficos, q deja agujeros porque 2 4 = 1 2 ), entonces q definitivamente no es incontablemente infinito.
@Quincunx Tienes toda la razón. Simplemente suena intuitivamente muy extraño para mí. Por ejemplo, di que F : R R es una función Supongamos que F es continua excepto en un conjunto contable de puntos. Puede sentirse tentado a pensar que esta función "casi se comporta bien", ¿verdad? Resulta que existe una función que es discontinua precisamente en todos los puntos racionales. Llegados a este punto, concluiría que la función no es en absoluto “casi buena”, sino más bien patológica. Después de todo, el conjunto de discontinuidades de F es denso!
Supongo que la única característica de q Lo que me desconcierta es que, de alguna manera, no me parece "correcto" que un conjunto contable sea denso en uno incontable.

La Hauptvermutung establece que esencialmente solo hay una estructura PL en una variedad. Más precisamente, establece que dos triangulaciones cualesquiera tienen una subdivisión común. La razón por la que esto parece "obviamente cierto" es que puede tomar ambas triangulaciones y superponerlas una encima de la otra, subdividiendo la variedad en un grupo de celdas y luego tomando la subdivisión baricéntrica para obtener una triangulación. Resulta que esto es falso y se necesitan algunas invariantes bastante sutiles para detectarlo. El problema con el argumento que di es que una triangulación podría ser muy salvaje con respecto a la otra (fractalmente ondulada) para que su unión no subdivida la variedad en una buena colección de celdas.

¿Para qué tipo de variedades falla?
@MJD Como recuerdo, hay un invariante del grupo fundamental (llamado grupo Whitehead) que se define en términos de matrices sobre el anillo del grupo, y esto debe ser distinto de cero para que la variedad tenga la posibilidad de ser un contraejemplo. Esto descarta muchos grupos fundamentales, incluido el grupo trivial y Z . Ha pasado un tiempo desde que he pensado en esto.

La paradoja de Stein es para mí la noción matemática más desconcertante que he conocido (aunque no soy matemático), principalmente porque no es un "artefacto" matemático, pero su falta de intuición conlleva consecuencias de error muy tangibles.

Teorema: (falso)

No se puede hacer nada mejor que la regla de decisión ordinaria para estimar la media de una distribución gaussiana multivariada bajo el error cuadrático medio.

En otras palabras, los fenómenos completamente independientes se pueden combinar para obtener un error de estimación conjunto más bajo.

¿Qué pasa con esto?

R y R 2 no son isomorfos (como los grupos abelianos con adición).

Cae dentro de la categoría de "Tomemos la base Hamel de R ...", pero me gusta mucho.

"Una secuencia de números en la que cada número es mayor que el anterior, eventualmente siempre superará un valor dado L".

Su respuesta es esencialmente equivalente a esta .
Ahh, buena captura, pero creo que la mía es más sencilla de expresar.

Realmente quiero que lo siguiente sea cierto:

Teorema: Sea S un subconjunto de un espacio vectorial. Si S es linealmente independiente por pares (lo que significa que cada { v , w } S es linealmente independiente) entonces S es linealmente independiente.

Y sin embargo, es falso. Por ejemplo,

{ v , w , v + w }
Si S solo tenía dos elementos, entonces ganamos por defecto. En cualquier caso, los estudiantes tienden a creer esto. Quiero decir, es álgebra lineal, el principio de superposición debería aplicarse, ¿verdad? Algo es la suma de sus partes, la independencia lineal engendra independencia lineal... muy seductor, muy equivocado.

¿Existe un espacio vectorial donde todos los pares de vectores sean linealmente independientes? ¿Quizás te refieres a una base de espacio vectorial? ¿O es la combinación de espacio vectorial y una base para un espacio vectorial una práctica común?
@DanielV, no es mi significado, lo reformularé, quise decir que si cada par de vectores en S es linealmente independiente... gracias por el comentario.
Me gusta tu ejemplo, solo quería asegurarme de que no lo estaba malinterpretando. Si mal no recuerdo, veo "espacio vectorial" para referirme al tramo de vectores base, por lo que dos conjuntos de vectores base diferentes pueden tener el mismo espacio vectorial. Para mayor claridad, probablemente habría dicho "Si cada par de vectores en S es linealmente independiente, entonces S es un conjunto de vectores linealmente independientes". Sin embargo, siento criticar, creo que incluso una descripción gráfica de esta afirmación podría sorprender a algunos estudiantes, así que creo que es un buen ejemplo.

Una de las primeras veces que me sorprendieron equivocandome en algo tan obvio fue creer:

abs(x) nunca es igual a -x

Por supuesto abs(x)se define como -xparax < 0

Un no-teorema relacionado que mis estudiantes quieren creer es que a 2 = a . (Por otra parte, tengo estudiantes que quieren creer que a + b = a + b .)
@TobyBartels dependiendo de cómo definas las cosas a 2 = a podría ser parcialmente cierto. Es a 2 el inverso (de varios valores) de X X 2 en a 2 , o es el único número positivo b tal que b 2 = a 2 ? La gente (de forma molesta para mi gusto) habla de ello en ambos sentidos.
@CharlesHudgins: Eso es cierto, aunque es un poco complicado de usar = con operaciones multivaluadas.
@TobyBartels Por eso, pedagógicamente hablando, estoy a favor de no enseñar a los estudiantes a resolver X 2 = 9 "tomando la raíz cuadrada de ambos lados" y luego golpeando en un ± . 1) Los estudiantes siempre olvidarán el ± . 2) Da la falsa impresión de que simplemente puede "deshacer" funciones no inyectivas. En su lugar, deben hacerse para reorganizar a X 2 9 = 0 y factorizar y luego resolver, para que entiendan por qué hay dos respuestas. Dicho esto, no aguantaré la respiración. Los cambios pedagógicos tardan una eternidad en abrirse camino en el plan de estudios de matemáticas.

Creo que esto no está cubierto en ninguna de las otras respuestas (aunque, sin duda, hay muchas). La paradoja de Simpson está cerca, pero creo que esta es diferente y algo más fácil de entender:

Si X se correlaciona positivamente con Y , y Y se correlaciona positivamente con Z , entonces X se correlaciona positivamente con Z .

En otras palabras, la correlación positiva es transitiva. Creo que es bastante intuitivo, pero falso.

Es cierto, sin embargo, que si ambos X y Z se correlacionan positiva y perfectamente con Y , entonces X está positiva y perfectamente correlacionada con Z .

"Obviamente"

( X y ) z = X y z
para X , y , z C tal que se definen las expresiones dadas.

Alguien tiene que decirle esto a los autores de libros de texto de álgebra elemental de nivel universitario. Estos afirman consistentemente que como un teorema cuando x es real e y y z son racionales.

Algo que solía ser seducido por mi inmadurez matemática (que lamentablemente aún existe):

Suponer que PAG norte son una familia de sentencias indexadas por norte norte y podemos asignar significado a PAG . Entonces sí PAG norte es cierto para todos norte norte , entonces PAG es cierto también.

Todo eso depende de la precisión con la que defina el "significado" de PAG .
"Supongamos que... podemos asignar significado a PAG ." Asumo que podemos.
En ese caso, pido un ejemplo donde el PAG se define "correctamente" pero en realidad es falso.
El ejemplo que tenía en mente cuando escribí esto era PAG norte : la suma de norte funciones diferenciables es diferenciable; PAG : la suma infinita de funciones diferenciables es diferenciable.
¿Qué tal la afirmación de que 1/n > 0, que se cumple para cualquier número entero n pero falla en el infinito?
@JónÁskellÞorbjarnarson Mark diría que 1 / > 0 no tiene sentido porque 1 / no está definido. Un ejemplo más fácil es simplemente una secuencia de racionales que convergen en un irracional.
@Jp: Claramente podemos asignar significado a 1 / . Incluso es útil. Solo es "no definido" en el mismo sentido que una suma ordinaria de infinitas cosas no está definida, o que para una familia PAG norte con norte norte , PAG no está definido.
@Hurkyl Estoy de acuerdo y estaba tratando de discutir desde la posición de Mark.
Esta es la Ley de Continuidad de Leibniz, ¿verdad?
@TannerSwett Siguiendo un rápido Google, creo que lo es.

El teorema de Cauchy implica que:

si uno hace un modelo físico de un poliedro convexo conectando placas rígidas para cada una de las caras del poliedro con bisagras flexibles a lo largo de los bordes del poliedro, entonces este conjunto de placas y bisagras formará necesariamente una estructura rígida.

Sin embargo, hay contraejemplos si permite un poliedro general (no convexo).

EDITAR: El contraejemplo que tenía en mente es incorrecto; He hecho otra pregunta para tratar de aclarar el asunto de una forma u otra. De todos modos, creo que es seguro decir que esto ya no es obvio. Pero continuaré y actualizaré (o eliminaré) mi respuesta en consecuencia una vez que tenga un poco más de claridad.

Aquí hay un ejemplo topológico que requiere un poco de reflexión para falsificarlo: aproximadamente, 'por cada curva que no se cruza entre dos esquinas opuestas de un cuadrado, hay una curva entre las otras dos esquinas que solo la cruza una vez'. Formalmente:

Dejar F : [ 0 , 1 ] [ 0 , 1 ] 2 ser una curva que no se corta a sí misma con F ( 0 ) = ( 0 , 0 ) , F ( 1 ) = ( 1 , 1 ) , y F ( t ) ( 0 , 1 ) 2 para t ( 0 , 1 ) . Entonces existe una curva que no se corta a sí misma gramo : [ 0 , 1 ] [ 0 , 1 ] 2 con gramo ( 0 ) = ( 1 , 0 ) , gramo ( 1 ) = ( 0 , 1 ) , y gramo ( t ) ( 0 , 1 ) 2 para t ( 0 , 1 ) tales que hay únicos t 0 y t 1 con F ( t 0 ) = gramo ( t 1 ) .

Esto parece obvio (al menos para mí) a primera vista, e incluso a segunda vista, el ejemplo del teorema de la curva de Jordan sugiere que debería ser cierto; después de todo, obtenemos un 'lado izquierdo' y un 'lado derecho' de nuestra curva por el JCT, ¿y el teorema de Schoenflies no significa que deberíamos poder encontrar una aplicación inversa de nuestra curva al círculo? Pero es falso; hay curvas F ( ) que no puede ser intersecado solo una vez por cualquier curva gramo ( ) . Encontrar un contraejemplo es un buen ejercicio...

¿Está seguro? Creo que puedo probar que tal g siempre existe.
@studiosus ¡Me encantaría ver tu prueba! Tenía bastante confianza en mi ejemplo, pero pensándolo mejor, estoy dispuesto a creer que se puede falsificar. Puede que no haya estado considerando lo suficientemente degenerado gramo ( ) para ir con mi canónicamente degenerado F ( ) ...
No hay lugar para una prueba en los comentarios, ya que es largo. Haz una pregunta aparte.
Hm, diría que se puede mostrar como un corolario del teorema de Jordan-Schoenflies (del cual no conozco una prueba).
Estoy de acuerdo con @Alexey. Añadir una curva desde ( 1 , 1 ) a ( 0 , 0 ) extender F a una curva cerrada F ~ eso ( 0 , 1 ) está dentro y ( 1 , 0 ) Está afuera. Encuentre un homeomorfismo Φ : R 2 R 2 del avión que mapea F ~ al círculo unitario (por el teorema de Jordan-Schoenflies). Dibuja un segmento recto que conecte las imágenes de ( 0 , 1 ) y ( 1 , 0 ) (esto cruza el círculo unitario exactamente una vez). La imagen previa de este segmento (es decir, su imagen bajo Φ 1 ) es la curva deseada gramo . (¿Bien?)
@mjqxxxx: No, eso no funciona: es posible que el segmento de línea recta no esté contenido en Φ ( [ 0 , 1 ] 2 ) (ya que no sabe cómo se ve la imagen del resto del límite del cuadrado unitario). Uno necesita más trabajo para obtener una prueba real.
@studiosus: ¡Tienes razón! No me di cuenta de eso. Y no hay razón para que la imagen del interior de la unidad se cuadre debajo Φ debe ser del todo agradable. Tal vez esa es la arruga aquí.
Esto es un poco como una puñalada en la oscuridad, pero ¿puede haber tal ag si f es una curva que llena el espacio?
@ChristopherCreutzig: Ambos mapas f y g son 1-1, por lo tanto, no pueden llenar espacios.
Aquí hay un teorema más fuerte (de enderezamiento): Sea S ser una superficie con alguna triangulación fija (digamos, un plano triangulado) y h : GRAMO S una incrustación topológica de un gráfico finito. Entonces h es isotópica a una incrustación lineal por partes h : GRAMO S . Esto reduce los problemas del tipo anterior a problemas lineales por partes que son fácilmente solucionables. Sin embargo, se puede dar una solución más directa usando solo Schoenflies y un poco de topología algebraica.
Sí, todos mis contraejemplos en realidad fallan en la continuidad, no había estado prestando suficiente atención (tenían 'líneas de acumulación', estilo de curva sinusoidal de topólogo, pero por supuesto eso rompe la suposición). Borraré este teorema no falso en un momento y haré mi ejemplo como una pregunta adecuada, ¡ya que la prueba me parece muy interesante!
@studiosus (¡Puedes encontrar esa pregunta en math.stackexchange.com/questions/821025/… si quieres responder la pregunta correctamente allí!)

Lo siguiente es obviamente falso, pero en realidad es cierto, como se muestra en el artículo de Wikipedia sobre los conjuntos de Vitali .

Existe una colección contable { V norte } de subconjuntos del círculo unitario tal que:

  1. Cualquiera de los dos distintos V norte son disjuntos.
  2. Cualquier V norte puede obtenerse de cualquier otro mediante una rotación.
  3. La unión de todos los V norte es todo el circulo.

Todos V norte debe tener, de la propiedad dos, el mismo "tamaño" (para cualquier definición razonable de "tamaño"), pero si el hecho anterior fuera cierto, la suma de sus tamaños (iguales) sería el tamaño del círculo (positivo, pero finito). Pero si el tamaño fuera cero, la suma debería ser cero, y si el tamaño fuera positivo, la suma debería ser infinita.

Una consecuencia de esto es que lo siguiente es falso (aunque a todos nos gustaría que fuera cierto):

existe una funcion m que, dado un subconjunto acotado de R , te dice su "tamaño". Precisamente:

  1. Si A R está acotado, entonces m ( A ) [ 0 , [ .
  2. Si { A norte } norte norte es una secuencia de subconjuntos disjuntos acotados de R (eso es, A norte A metro = cuando sea norte metro ) con unión acotada (es decir, A norte está acotado), entonces m ( A norte ) = m ( A norte ) .
  3. Si A está ligado, X es un número real, y definimos A + X = { a + X : a A } , entonces m ( A + X ) = m ( A ) .
  4. m ( [ 0 , 1 ] ) = 1

En efecto, reescribiendo el primer hecho intercambiando el círculo por el intervalo semiabierto [ 0 , 1 [ e intercambiando rotaciones por turnos cíclicos "mod 1 ", nos damos cuenta de que si m satisface las tres primeras condiciones anteriores, entonces m ( A ) = 0 para todos A .

Si consideras un número finito de conjuntos V norte , y tomemos un límite, parece plausible, no del todo obviamente falso.

La paradoja del cumpleaños

Si se seleccionan al azar 30 personas y tienen cumpleaños que se distribuyen de manera independiente (idéntica) uniformemente durante el año calendario, entonces la probabilidad de que dos (o más) de ellos tengan el mismo cumpleaños es aproximadamente 1 12 .

Antes de 1955 todo el mundo “sabía” que conocer el enésimo dígito decimal de π (y para cualquier otro irracional) era necesario conocer los dígitos anteriores. Un genio como Arquímedes (" Había más imaginación en la cabeza de Arquímedes que en la de Homero ": Voltaire) "lo sabía" muy bien como lo demuestra la Historia. Sin embargo, la fórmula Bailey-Borwein-Plouffe (fórmula BBP) terminó con este sagrado "conocimiento" durante siglos y ahora es posible conocer, por ejemplo, el dígito 33 sin conocer los precedentes.

En cuanto a la percepción intuitiva, es falso que una función numérica continua deba ser derivable al menos en un punto; es falso también que un pequeño cuadrado no pueda contener una curva de longitud infinita.

La fórmula BPP para pi no da dígitos decimales, sino dígitos hexadecimales.
Creo que "1955" aquí debe ser un error.
La idea de que "un pequeño cuadrado no puede contener una curva de longitud infinita" se vuelve menos "obvia" cuando las personas se dan cuenta de que cada diminuta célula de su cuerpo contiene hebras muy largas de ADN. (Por supuesto, el ADN no es infinitamente largo y la célula es tridimensional, por lo que esto no es una refutación de la idea "obvia", pero parece arruinar la obviedad).

Aquí hay algunas declaraciones "obvias" a las que se puede aplicar la paradoja de Richard :

  1. Para un predicado dado PAG , existe un conjunto S de X para cual PAG ( X ) es verdad. (La paradoja de Russell)
  2. El conjunto de los números enteros y el conjunto de los números reales tienen el mismo tamaño infinito. (Argumento diagonal de Cantor).
  3. Existe una formalización de la aritmética en la que todos los enunciados verdaderos son exactamente aquellos que son demostrables. (Teorema(s) de Gödel)
  4. Existe un programa de computadora (máquina de Turing) que puede determinar efectivamente si cualquier otro programa de computadora no se detiene. (Problema de detención)
@MJD Sí, hice +1 :) Tu insistencia está bien justificada.
Creo que el n.° 4 no solo no es obviamente cierto, sino obviamente falso, porque si fuera cierto, entonces no tendríamos que ejecutar nuestros programas de computadora para averiguar qué hicieron, sino que podríamos usar este procedimiento hipotético para decírnoslo. , sin ejecutarlos. Pero entonces, ¿cuál sería el punto de ejecutar los programas? El universo simplemente no funciona de esa manera; si quieres saber qué sucede, entonces, al menos algunas veces, solo tienes que probarlo y ver.
También # 2 ya se menciona en mi respuesta.
@MJD Mis disculpas, acabo de corregir un error tipográfico en el n. ° 4 en el que reemplacé "detiene" con "no se detiene" al final de la oración.
No importa; mi comentario es el mismo de cualquier manera. Tengo una publicación de blog redactada que afirma que el teorema de Rice es el teorema más obvio en matemáticas, y la indecidibilidad del problema de detención es un caso especial de eso.
@MJD No veo la esencia de tu punto. Parte del problema de la detención es que si un programa no se detiene, ¿cómo lo sabremos? No podemos simplemente "probarlo y ver": ¿cuánto tiempo debemos esperar antes de decidir que no se detiene? Este es el problema esencial. Este tipo de declaración puede haber sido más "obvia" a principios del siglo XX, cuando la automatización parecía ser el nombre del juego, también reflejado en los objetivos de Russell y Whitehead al formalizar las matemáticas. Es genial que ahora sea obviamente falso para ti, pero yo diría que es debido a resultados como este que ahora estamos tan iluminados.
Déjame ponerlo de otra manera. Si el teorema de detención fuera falso, entonces habría un método para decir, con bastante rapidez, cuál sería el resultado de cualquier cálculo, incluso si ese resultado se encontrara en un futuro lejano o infinito. Pero sabemos que predecir el futuro lejano es imposible; está en la misma categoría de fantasía que los duendes y los unicornios. Entonces, ¿por qué deberíamos sorprendernos de que, de hecho, no haya ninguna forma de saber ahora qué hará una máquina de Turing en el futuro eventual? Usted afirma que es obvio (pero falso) que podamos predecir correctamente el futuro infinitamente lejano. ¿Por qué?
No se trata de predicciones generales del futuro; esto tiene que ver con el tiempo solo en la medida en que el infinito contable se puede asignar a pasos discretos de la máquina de Turing que tienen lugar en el tiempo. Por ejemplo, una máquina de Turing que se detiene cuando encuentra a , b , C , norte Z + : a norte + 2 + b norte + 2 = C norte + 2 sabemos que no se detiene gracias a la demostración de Wiles. Si el teorema de detención fuera falso, entonces una "máquina de prueba de teoremas" basada en una formalización de cualquier área de las matemáticas escupirá eventualmente todas las declaraciones verdaderas. Esto era "obvio" para Russell, Whitehead e incluso Hilbert hasta la década de 1930 gracias a Gödel, Turing et al.
@MJD Si el teorema de detención fuera falso, no implica que podamos saber el resultado de cualquier cálculo. Simplemente implicaría que existe un algoritmo. Todavía hay una brecha sustancial entre la existencia de tal algoritmo tu , y tanto (1) nuestra capacidad física para correr tu (puede requerir más memoria que la cantidad de átomos que se sabe que existen) y (2) lo que es más importante, nuestro conocimiento de que un algoritmo dado es de hecho tu . (1) y (2) no se siguen automáticamente de la falsedad del teorema de detención. Las razones por las que es cierto son más sutiles de lo que lo estás enmarcando.
@MJD Estoy de acuerdo con Matt. Incluso si el teorema de la detención fuera falso, no significa que el algoritmo sería práctico en absoluto y, por supuesto, la razón exige que sea tremendamente poco práctico. Como señala Matt, una máquina de prueba de teoremas puede escupir todas las afirmaciones verdaderas si el teorema de detención fuera falso; dado que es cierto, dicho programa solo arrojará declaraciones demostrables y no puede decirle que un teorema dado no es demostrable a menos que espere una eternidad. Pero incluso eso suena genial: ¿por qué no ejecutamos máquinas de prueba de teoremas para escupir todas las declaraciones comprobables y sacar a los matemáticos del negocio?

Hay muchos ejemplos en la teoría de grafos (extrema), donde un argumento obvio muestra que una declaración es verdadera, excepto que hay una serie de pequeños contraejemplos que son fáciles de pasar por alto.

Considere la siguiente afirmación: Sea GRAMO ser un gráfico con norte vértices y el mayor número de aristas sujeto a la condición de que GRAMO no contiene un par de aristas disjuntas (es decir, k 2 + k 2 ). Entonces GRAMO es una estrella (es decir k 1 , norte 1 ).

Esto es obviamente cierto, si lo piensas por un momento. Pero para norte = 3 , una mejor solución es tomar GRAMO = C 3 . Y para norte = 4 , tomando C 3 además, un vértice aislado es tan bueno como tomar k 1 , 3 .

Un orden lineal puede reconstruirse de forma única (hasta el isomorfismo ) a partir del conjunto de tipos de orden de sus segmentos iniciales propios .


Actualización: incluso si conocemos la cardinalidad del orden lineal y sabemos que no tiene un elemento máximo, este "teorema" aún no se cumple.

Todos los infinitos son del mismo tamaño.

Pero el teorema de Cantor muestra lo contrario.

Un subgrupo de un grupo generado finitamente puede no generarse finitamente y hay hasta isomorfismo como máximo dos pag q grupos, donde pag y q son primos

Para fortalecer tu ejemplo, pag   pag r i metro mi 1 / pag diverge

La hipótesis del continuo también parece tener una respuesta en ZFC, lo cual no es así.

En otra página, los matemáticos pensaron que los campos ciclotómicos "obviamente satisfacen el teorema de factorización única", lo que lleva a algunos intentos de prueba falsa del último teorema de Fermat.

A continuación, se podría pensar que la "trisección de ángulos" es posible o que cualquier conjunto de funciones analíticas { F α } , tal que por cada z C , el conjunto { F α ( z ) } es contable, ella misma tiene que ser contable.

Estos son solo algunos ejemplos aleatorios que me vinieron a la mente y, dado que el término "obvio" es subjetivo, es muy posible que no esté de acuerdo con los elementos de mi lista. Supongo que depende en gran medida de tu formación matemática.

No estoy de acuerdo con que haya algo intuitivamente obvio sobre la hipótesis del continuo, y encuentro aún menos que sea intuitivamente obvio sobre lo que se puede o no probar en ZFC, que es tan complejo que tienes que estudiar mucho para entender cuáles son los axiomas. dicho.

Si comienza a pensar en la rigidez de las conchas delgadas en R 3 , te encuentras rápidamente con una serie de resultados contradictorios.

Por ejemplo, es obvio que una capa esférica es ( C 2 ) rígido, y esto es de hecho cierto. Una superficie lisa, cerrada y compacta con una curvatura gaussiana positiva en todas partes es igualmente rígida. Uno podría imaginar que estos resultados se generalizan a

  • Cualquier superficie cerrada;
  • Cualquier superficie cerrada con curvatura gaussiana positiva en todas partes pero en un número finito de puntos;
  • Cualquier superficie con límite con curvatura gaussiana positiva en todas partes;

y todo esto es falso.

Además, después de pensar en los reflejos, o de pinchar y empujar una pelota de ping-pong, es intuitivamente obvio que una capa esférica no es C 0 rígido. Pero realmente no puedes "ver" ninguna diferencia entre un C 1 y un C 2 deformación de la esfera, por lo que seguramente la esfera es C 1 ¿rígido? Lejos de ello, dada cualquier superficie cerrada arbitraria que sea topológicamente una esfera, y la distancia ϵ , es posible C 1 -insertar isométricamente una esfera ϵ -¡Cerca de la superficie objetivo!

El siguiente es un ejemplo muy conocido, aunque probablemente un poco fuera del mundo de las matemáticas, más bien de la física. Muchas personas considerarían 'intuitivamente' que lo siguiente es cierto:

The heavier the object, the faster it falls down.

De hecho, la historia dice que se suponía que esto era de conocimiento común hasta que Galileo Galilei lo refutó (según cuenta la historia, al dejar caer dos bolas de la torre en Pisa, lo que nunca sucedió).

Una de las primeras clases de física que mucha gente tiene (estoy hablando de la escuela primaria aquí) tiene como objetivo demostrar que este teorema es falso, y en realidad todo cae con la misma aceleración (ignorando la resistencia del aire) independientemente del peso.

Todo cae con la misma aceleración no a la misma velocidad... gran diferencia.
La afirmación de Aristóteles era en realidad que los objetos caen con una velocidad proporcional a su peso , lo cual es mucho más específico y mucho menos obvio. Es increíble que la civilización occidental se haya tragado este bicho raro durante tanto tiempo.
@MJD Universalmente, las personas tienden a creer cosas según la popularidad en lugar de la evidencia o la experiencia. Esto es tan cierto hoy como lo fue en el pasado, y los estudios muestran que (contrariamente a la intuición) las personas bien educadas son mejores para hacerlo que las personas con poca educación.
Me burlo de esta idea en mis lecciones cuando reviso las respuestas de varios estudiantes y elijo la respuesta del modo, y anuncio: "Esta debe ser correcta, porque como todos sabemos, ¡la democracia es el camino hacia toda la verdad!"
Pero un globo de gas es más ligero (en relación con su volumen) que, por ejemplo, una roca e incluso se eleva en lugar de caer.

No es exactamente un teorema, pero engaña a todos los recién llegados a las matemáticas:

mi = límite norte ( 1 + 1 norte ) norte

( 1 + 1 / ) es 1 , obviamente. y 1 a la potencia de obviamente sigue siendo 1.

No, es 2.718...

Creo que el cartel dio este ejemplo en la pregunta, pero es bueno.
No, 1 es una forma indeterminada.

Aquí hay una proposición, cuando se pone en términos sencillos, una persona promedio afirmaría que es verdadera.

Todo subconjunto de números reales tiene una medida.

¿Cómo puede ser esto falso, cuando marcas una región, digamos en dos dimensiones, por supuesto, tiene un área? A menos que haya construido un conjunto Vitali en algún momento, tendemos a pensar que el concepto de longitud/área/volumen debe extenderse a todos los subconjuntos posibles.

Aquí hay otra proposición falsa.

Axioma de Determinación

Si estamos jugando un juego infinito de dos jugadores donde creamos un número real en [ 0 , 1 ] eligiendo dígitos decimales por turnos y uno de nosotros trata de colocar el número resultante en un conjunto de pagos predeterminado que ambos conocemos y el otro intenta evitarlo, ¿cómo puede ser que haya un juego en el que ninguno de los dos tenemos una estrategia ganadora? Ambos tenemos información completa sobre el conjunto de pagos, qué números evitar y qué números acertar, uno de nosotros debería poder idear una estrategia. Bueno, lamentablemente no.

Ambas proposiciones son inconsistentes con el axioma de elección, con el cual puedes construir los contraejemplos que no se "comportarán bien".

Realidad: La última proposición implica la primera. (en ZF, con el que se cree que AD es consistente).

Parece un poco exagerado llamar a AD "falso". AC no está en ningún plano epistemológico superior.
¿Cómo pondrías "cada conjunto de números reales tiene una medida" en términos simples, para que una persona promedio afirme que es cierto? Asegúrese de incluir la aditividad contable en la formulación de su lego, ya que eso es necesario para que la proposición sea falsa.
@bof: El concepto de medida se origina en nuestra noción intuitiva de longitud/área/volumen. Dudo que una persona promedio se oponga a eso. Elija un cuadrado unitario y coloree un subconjunto de él, debe tener un "área" comparable a las áreas de formas básicas. O, pregunta esto, si lanzo un dardo a este cuadrado, ¿crees que hay una probabilidad bien definida de que golpee la región coloreada? En cuanto a la aditividad contable, ¿nuestra intuición sobre el "área" no incluye esto? Si te doy piezas disjuntas que puedes etiquetar por naturales, claro que el área de la unión es la suma de las áreas?
@KevinCarlson Sin estiramiento, pero de uso común. "Falso" (sin calificadores, como "falso en este o aquel modelo") es una abreviatura de "falso según la interpretación estándar", que en este caso significa según el conjunto estándar de axiomas, a saber Z F C , incluida la elección.

Ejemplo contrario a la intuición

    D v F ( a ) = 0 , v , a F     continuo .

Por D v Quieres decir F ( v ) , como en el v -ésima derivada?

Pruebas de geometría hechas de manera informal dibujando figuras en la pizarra. Entonces pasa por alto los axiomas de la geometría euclidiana, pretende que no necesita invocarlos ya que las figuras dibujadas parecen suficientes. Sin embargo, en la gravedad de la Tierra, la geometría euclidiana es solo una aproximación.

Infinidad de términos siempre tienen una suma igual a infinito.

¿ Quieres decir infinitamente muchos términos? Si no, ¿qué es un "término infinito"?
Eso es infinitamente muchos términos. :)
Oye, no creo que eso sea cierto porque una progresión geométrica tiene infinitos términos y aun así converge.

Alguien más mencionó "hay más números racionales que enteros". En la misma línea, me costó mucho aceptar que

Hay más números enteros que números reales entre 0 y 1

Es falso. Quiero decir, ahora lo entiendo, pero intuitivamente me parecía muy mal antes de estudiar los números transfinitos.

Otro ejemplo de que 'obvio' no es cierto es la paradoja del tiempo de espera del autobús .

Si el tiempo medio entre dos autobuses consecutivos que llegan a una estación de autobuses es METRO , uno debe esperar que el tiempo medio que debe estar esperando en la estación antes de que llegue el próximo autobús también sea M. Pero esto no es cierto; y dependiendo de la distribución de la hora de llegada del autobús en particular, tendrá que esperar un tiempo METRO METRO

Para mí, un buen ejemplo de todas las "evidencias" que sugieren que era cierto es

π ( X ) < li ( X )

hasta que Skewes demostró que π ( X ) li ( X ) cambia de signo infinitamente a menudo

La pregunta pide declaraciones obvias pero falsas. ¿Estás afirmando en serio que π ( X ) < li ( X ) ¿es obvio?
difícil decir si se supone que eso es obvio... ¿Cuáles son π ( X ) y li ( X ) ?
π ( X ) es la función de conteo primo , yo i ( X ) es la integral logarítmica. Ramanujan afirmó la desigualdad. Littlewood demostró en 1914 que el signo cambiaba infinitamente a menudo y Skewes dio un límite superior para el primer cambio.

Aquí hay una cosa que comúnmente se cree que es cierta, pero que es terriblemente incorrecta en muchos aspectos:

There is a notion of mathematics where we can say things are 
"actually true" or "actually false".  

Un ejemplo de cometer este error: el OP. Otros ejemplos: las múltiples respuestas.

Hay varias razones por las que esto está mal. Primero, en el sistema que la mayoría de los matemáticos asumen cuando no son explícitos, no tenemos un modelo estándar (no tenemos modelos dentro de ese sistema porque cualquier modelo mostraría el sistema consistente que sabemos que no podemos mostrar en ese sistema a través de bla, bla, Godel, bla). Sé que no quieres detalles, solo explicar a lo que me refiero). La verdad y la falsedad son semánticas: existen en modelos y, por lo tanto, sin uno, no hacemos afirmaciones de verdad o falsedad.

Pero también, las matemáticas no son "el sistema que la mayoría de los matemáticos asumen cuando no son explícitos", es la formalización en general. Hay muchos sistemas seriamente investigados por matemáticos que hacen numerosas derivaciones "contrarias a la intuición". Por ejemplo, todos estos son obvios e incorrectos en diferentes sistemas:

  • Una afirmación no puede ser a la vez verdadera y falsa. (En la lógica paraconsistente, las declaraciones pueden ser tanto verdaderas como falsas y el sistema no colapsa hasta convertirse en algo trivial; de hecho, varios dialeteístas argumentan que este es un sistema lógico mucho más preciso para el razonamiento del mundo real).
  • Hay funciones totales discontinuas. (En varios sistemas constructivos no es posible demostrar la existencia de funciones totales discontinuas. Algunos incluso son lo suficientemente fuertes como para demostrar que todas las funciones totales son continuas).
  • Todo conjunto infinito A tiene la misma cardinalidad que AxA. (Esto no es necesariamente cierto en los sistemas sin el axioma de elección. Tarski intentó publicar su resultado sobre esta implicación y fue rechazado tanto por Frechet como por Lebesgue. Frechet pensó que el artículo era obvio y conocido y no tenía ningún mérito matemático. Lebesgue pensó que tanto el axioma de elección como la implicación de este eran incorrectos, por lo que el artículo no tenía mérito matemático).

Solo menciono estos ejemplos, no como respuestas al OP, sino simplemente para ilustrar mi respuesta real de que la pregunta en sí misma demuestra una suposición extremadamente común en matemáticas que, de hecho, es incorrecta.

EDITAR

Esta es un área que creo que a menudo es un lugar de malentendidos comunes, y la discusión en los comentarios deja en claro que debo elaborar. Las matemáticas modernas separan los dominios sobre los que hacemos declaraciones en sintaxis y semántica.

Sintaxis

La sintaxis es la teoría: el lenguaje formal, los axiomas especificados como oraciones en el lenguaje formal y algunas reglas metalógicas de inferencia. En la sintaxis, hablamos de oraciones, proposiciones, términos, derivaciones y pruebas. Es un lugar de manipulación de símbolos.

Semántica

La semántica es el modelo, es el significado que atribuimos a los enunciados de la teoría. Una interpretación de una teoría es un modelo que asigna a cada fórmula de la teoría un valor de significado, típicamente la verdad. La verdad es semántica y es específica de un modelo.

El problema"

Un modelo es una interpretación consistente del significado de verdad de una teoría. Si una teoría tiene un modelo, casi trivialmente se ha demostrado que es consistente. Pero... es bien sabido que una teoría lo suficientemente fuerte como para expresar la diagonalización de Gödel nunca puede probar su propia consistencia. Para estas teorías, nunca tendremos un modelo y no podemos hacer afirmaciones sobre el significado de ninguna fórmula.

En estas teorías, es incorrecto hablar de verdad o falsedad. No tenemos un modelo que dé sentido a eso. Nunca tendremos un modelo.

Eso no es realmente un problema. Durante siglos, los matemáticos habían combinado vagamente la derivación y la verdad y las habían discutido principalmente como una sola cosa. La derivación y la prueba se consideraban la parte importante de las matemáticas y la formalización. Todavía tienes eso.

Además, es perfectamente significativo derivar resultados que digan " si esta teoría es consistente y tiene un modelo, entonces...". La teoría de modelos ha estado haciendo eso durante casi un siglo.

¿Y los predicados de verdad?

Pero la gente parece querer más. Quieren hablar de la verdad, ya que es una forma de significado que ocupa un lugar especial. A menudo hacen todo lo posible para tratar de seguir asignando verdad y falsedad. Un enfoque común es formar predicados de verdad: predicados en la sintaxis que tienen la propiedad de que afirmar el predicado en una fórmula corresponde a afirmar la validez del enunciado (que es verdadero en todos los modelos).

Tenga en cuenta el cambio: un predicado de verdad es sintáctico. Todavía no estamos hablando de verdadero o falso aquí: el contexto de su uso sigue siendo si las declaraciones que incluyen el predicado "son derivables" u "obtienen". Las teorías pueden tener múltiples modelos: la mayoría de las teorías no son categóricas solo de cosas como Löwenheim-Skolem, por lo que los predicados no pueden hablar sobre la verdad. Pueden hablar de validez, y eso es realmente lo que está pasando aquí, pero incluso eso es extremadamente problemático.

Las teorías incompletas en realidad no pueden derivar nada acerca de la validez de la teoría total. Y de hecho, aquí es donde entra el teorema de Tarski sobre la indefinibilidad y se demuestra que tal predicado en realidad no existe. Así que otros siguen con una jerarquía y extensiones de reflexión de la teoría base, buscando alguna aproximación de un punto fijo para la validez.

Pero esto en realidad no compra nada que tenga que ver con la comprensión de la verdad. No puede. No hay nada que puedas hacer para alcanzar la verdad porque no puedes saber si la teoría es consistente o no y si la verdad existe. Y ningún intento de ir más allá de la derivabilidad en realidad da un predicado que se puede usar y decir "esto es cierto". El predicado solo es útil para decir "esto es demostrable".

Pero ya hay predicados de demostrabilidad, y esa investigación es mucho más rentable. Los predicados de verdad son oráculos sin voz. No ayudan a nadie a hacer afirmaciones sobre la verdad. Son simplemente reformulaciones de "si supiéramos que X es consistente, y tuviéramos una visión platónica que pudiera ver los valores de verdad en todos los modelos, y pudiéramos cotejar las infinitas posibilidades y ver las validezes ocultas para siempre, entonces este predicado se aplicaría a esto". clase de enunciados estaría de acuerdo con aquellas afirmaciones que son válidas". Pero si tuviéramos esa vista sobrenatural, podríamos decir más fácilmente "oye, eso es cierto en ese modelo, y eso es falso allí". Sin eso, podemos usar el predicado para decir "la verdad se conserva en esta derivación". Lo cual no agrega nada.

Un predicado de verdad no habla de la verdad. Es irrelevante al punto.

Entonces...

Así que... la vida continúa. Todo mi objetivo al publicar esta respuesta fue ilustrar que la pregunta inicial estaba haciendo una suposición obvia común que en realidad es incorrecta. No debe hablar sobre la verdad en la teoría ambiental de uso común, solo hable sobre lo que es demostrable y estará bien. Si desea hablar sobre la verdad, asegúrese de especificar la teoría ambiental y es una en la que tales discusiones sean significativas. O hablar de modelos condicionales como lo hacen los teóricos de modelos.

Puede que no sea intelectualmente satisfactorio para algunas personas. Claramente, al momento de escribir esto, mi respuesta ha recibido 3 votos negativos y dos positivos, por lo que no le sienta bien a algunos lectores anónimos de un sitio web de matemáticas. Pero no hay nada controvertido sobre el punto. Se conoce desde hace casi 100 años y sigue siendo un error común.

Creo que la tercera no es ni obviamente verdadera ni obviamente falsa.
@AsafKaragila: Yo también. Lo pongo ahí porque hay una anécdota famosa de dos matemáticos que lo creían obvio, pero de los dos lados. Fue más para ilustrar el punto de que muchos cometen el error de categoría de pensar que los teoremas son "verdaderos o falsos" en lugar de "obtenidos en el sistema ambiental S". La verdad o la falsedad, de hecho, incluso la validez, no son realmente aplicables cuando su sistema ambiental no puede proporcionar una semántica sobre la cual hacer esas afirmaciones.
Su tercer párrafo, en su forma más precisa, es el teorema de indefinibilidad de Tarski. Pero siendo consciente de lo que es, todavía no estoy de acuerdo con los dos párrafos anteriores. En relación con otro sistema, puede hablar sobre la verdad, por ejemplo, la verdad aritmética se puede definir en ZFC (pero no en PA). La verdad en el universo teórico de conjuntos no se puede definir en ZFC, pero ¿y qué?
Si cree que existe un universo teórico establecido y la lógica de primer orden captura su verdad (lo que significa que los axiomas son verdaderos [sea lo que sea que esto signifique]), entonces podemos interpretar que ser demostrable/refutable como verdadero/falso. ¿Por qué tenemos que ser capaces de definir la verdad (formal) en el universo que suponemos que existe? Si le molesta hablar de la verdad y la falsedad, puede volver a leer la pregunta y todas las respuestas (que parece afirmar que son defectuosas) como "probablemente cierto" y "probablemente falso".
Una última observación rápida: la verdad formal en el universo de la teoría de conjuntos es realmente definible, pero no uniformemente. Lo que esto significa es que si pone un límite a la complejidad de sus fórmulas (por ejemplo, en la jerarquía de Levy), entonces puede definir la verdad de esas fórmulas (usando una sola fórmula que tiene una mayor complejidad). No poder hacer esto de un solo golpe (con una sola fórmula) no debería hacerte pensar que la verdad matemática no tiene sentido ni que verdadero/falso no tiene sentido.
@Burak: Mi comentario dio dos razones por las que los párrafos anteriores estaban equivocados. Sí, la primera fue la "coherencia estándar probada en la misma teoría para teorías lo suficientemente fuertes para PA" para señalar que simplemente asumir la teoría ambiental estándar era insuficiente para hacer esas afirmaciones. La segunda razón, sin embargo, fue señalar que si no asumes el ambiente estándar, entonces la noción de lo que es verdadero o falso cambia drásticamente según la teoría del ambiente. Ninguna de esas dos declaraciones es controvertida y cubren lo que estaba tratando de mostrar.
@ex0du5: Veo lo que estás tratando de mostrar. Por otro lado, el cambio en la teoría de fondo que afecta a la verdad formal no hace que la pregunta carezca de sentido, ni ninguna de las respuestas. Simplemente puede suponer que la pregunta del OP tiene lugar en ZFC como teoría de fondo (o alguna teoría en la que puede formalizar objetos de sus teoremas). Puede ser mi malentendido de su respuesta, pero interpreté lo que ha escrito como un enfoque extremadamente escéptico que utiliza fenómenos de incompletitud/indefinibilidad para rechazar cualquier tipo de noción semántica.
@Burak: Hay muchas formas de iniciar un programa para "arreglar esto". Pero es importante tener claro lo que esos programas están intentando. La demostrabilidad y la verdad están separadas y se han delineado claramente en las investigaciones metamatemáticas desde antes de Gódel. Podemos dar una modalidad a la demostrabilidad y hablar de relaciones accesibles, pero los términos a usar aquí son "derivables" u "obtenibles". Yendo por el otro lado: las definiciones de verdad no son verdad. En ambas discusiones, hay un error de categoría que confunde la sintaxis con la semántica. (continuación)...
@Burak: Una forma de ver mi respuesta es que estoy señalando "es una suposición común que se puede hablar sobre la semántica en ZFC en la forma en que los matemáticos se han desarrollado durante el siglo pasado", ya que claramente no se puede. Puedes tratar de dejar de lado la importancia de la semántica, puedes tomar la existencia de una semántica como una creencia, puedes sustituirla por definiciones sintácticas, pero nada da esa semántica en la forma en que se han desarrollado las matemáticas. De hecho, esos puntos fueron precisamente algunas de las razones que provocaron la necesidad de ser precisos sobre la verdad y la semántica.
@ex0du5: Ya veo. Entonces, ¿quiere señalar la diferencia entre la formalización de la verdad y la "verdad real" (lo que sea que eso signifique)? ¿Tengo razón? Si es así, estoy de acuerdo en que es importante tener en cuenta la distinción, pero dado que no hay forma de hacer metamatemáticas sin formalizar estos conceptos, discutir la diferencia parece una cuestión filosófica. El mismo problema se aplica también a los números naturales. ¿Es cierto que nuestros sistemas de axiomas son aritméticamente correctos, lo que significa que lo que prueban es "realmente" verdadero en "los" números naturales? No hay nada matemático que hacer contra esta pregunta.
@Burak: No estoy diciendo nada en contra de la formalización. Todo lo contrario, estoy diciendo que hay una clara descripción formal de la verdad que ha sido refinada y aplicada para el gran beneficio de las matemáticas. No hay filosofía en mi respuesta. La verdad es "la verdad en un modelo dado de una teoría dada". Asumir otro significado para la verdad ha sido un no-no durante un siglo, y hay razones bien conocidas por las que ese error llevó a los matemáticos por mal camino. Realmente estoy respondiendo a la pregunta original al dar una creencia "obvia" real que, de hecho, es incorrecta. No estoy tratando de ser sutil.
@ex0du5: Si aceptamos usar la noción tarskiana de verdad, entonces debo repetir mis tres primeros comentarios. Usted está diciendo en su publicación que no existe una noción de matemáticas en la que podamos decir "realmente cierto". Esta es la indefinibilidad de Tarski, nada más. Lo que digo es que si estás dispuesto a sacrificar la uniformidad, entonces existe una noción definible de "realmente cierto". Puedo darte una descripción inductiva de oraciones verdaderas, digamos en teoría de conjuntos, que tienen como máximo n cuantificadores ilimitados siempre que fijes n. (continuación)
@ ex0du5: Para aclarar nuestra falta de comunicación, ¿cuál es exactamente la declaración (en forma matemática ) que usted afirma que se cree que es obviamente cierta pero que es incorrecta en muchos aspectos?
El teorema de incompletitud de Gödel realmente me lleva a extrañas discusiones filosóficas después de una o dos cervezas, ya que establece que las reglas de las matemáticas no se basan en las leyes de la naturaleza y son solo herramientas 'inventadas' por nosotros: "Cualquier teoría generada de manera efectiva capaz de expresar la aritmética elemental no puede ser a la vez consistente y completa".
@Burak: amplié mi respuesta. Espero que aclare por qué los predicados parciales nunca pueden expresar la ontología de la verdad y cómo las construcciones sintácticas no hacen nada para abordar el punto de la semántica inaccesible de las teorías lo suficientemente fuertes para la construcción de Gódel. Si no, no sé qué decir, ya que esto es muy conocido y apenas controvertido, aunque entiendo que deja a algunas personas con una angustia existencial.
@aRestless: Creo que su comentario sobre las leyes de la naturaleza expresa una parte importante de por qué esto deja a tantos con una impresión tan fuerte. En la naturaleza, si hablamos completamente operativamente, podemos saber la respuesta a una consulta semántica simplemente "mirando" (realizando la operación o experimento). Toda la verdad es alcanzable. Entonces, ¿por qué no matemáticas? ¿En particular algo tan simple como la aritmética?
@ ex0du5: Su expansión de la publicación al menos aclaró dónde mi punto de vista difiere del suyo. Podemos inventar un predicado de verdad (formalizado) que nos diga si una oración (formalizada) es verdadera en un modelo o no. Estás diciendo que estos predicados no hablan de "verdad". Tiendo a pensar que sí, ya que este tipo de formalización (incrustar conceptos semánticos en la sintaxis) de la verdad parece ser la única manera de hablar matemáticamente de la verdad (porque así es como se han hecho las matemáticas durante mucho tiempo, sintácticamente, en un sistema formal).
@ ex0du5: un comentario adicional sobre su párrafo "el problema". Incluso en el caso de que nuestra teoría demostrara su propia consistencia (o, de manera equivalente, que tenga un modelo establecido), dado que está negando que los predicados de verdad hablen sobre la verdad, no podría hablar sobre la verdad de una oración. De lo contrario, todos sus problemas podrían resolverse mediante un axioma que afirme la existencia de un modelo de su teoría. Aparte de eso, no estoy seguro de que sea "bien conocido" que los predicados de verdad (o cualquier formalización de conceptos semánticos en la teoría de modelos) carezcan de sentido.
@ ex0du5 Usted escribió: "una teoría lo suficientemente fuerte como para expresar la diagonalización de Gödel nunca puede probar su propia consistencia". ¿Es esta afirmación "realmente cierta"?
@VladimirReshetnikov No lo sé. Pero sí sé que es demostrable en esas teorías.
@ ex0du5 Escribiste: "es demostrable en esas teorías". Aparentemente, esta es una afirmación universalmente cuantificada sobre ciertas teorías. ¿Es esta afirmación realmente cierta?
@ ex0du5 Supongo que no verificó manualmente la demostrabilidad en todas las infinitas teorías que está cuantificando. ¿Cómo puedes decir "Sé que es demostrable en ellos" sin atribuir ningún valor de verdad al enunciado que expresa demostrabilidad en una teoría? Saber algo por lo menos significa que crees que es verdad, ¿no es así?
@VladimirReshetnikov Lamento que mi respuesta haya sido alegre. Tomé tu pregunta inicial igual. Sin embargo, siendo honesto, creo que los únicos fundamentos que abordan seriamente esta cuestión son los constructivistas radicales en algún lugar entre el predicativismo y el ultrafinitismo, donde tal cuantificación no es válida. Personalmente, tengo mucha afinidad con los fundamentos computacionales de esas escuelas y su fuerte enfoque en la semántica, ya que creo que es el único enfoque del significado que puede funcionar. He tratado de evitar eso arriba y ceñirme a cosas ampliamente aceptadas, pero tal vez no pueda evitarlo.

La siguiente afirmación es incorrecta:

The inner angles of a triangle always sum to 180 degrees.

Si bien suena plausible que la suma de los ángulos sea una constante, en realidad es una propiedad del espacio. En el espacio euclidiano, los ángulos internos de un triángulo siempre suman 180 grados.

Pero es bastante obvio y es cierto.
@bjb568 Creo que Robert puede estar pensando en triángulos no planos.
@MJD Bueno, eso tiene más sentido, pero no es obvio...
Sí, parece obvio que la suma de los ángulos es una constante. Pero en realidad esto solo es siempre cierto para los triángulos planos.
No me parece obvio que los ángulos de un triángulo plano siempre tengan la misma suma. De hecho, parece asombroso, y estoy seguro de que podría encontrar otros lugares en este sitio web donde otras personas dijeron que parecía asombroso.
Por ejemplo, esta respuesta altamente votada comienza "Me pareció completamente sorprendente que los ángulos en un triángulo siempre sumaran 180 grados".
Veo que la corrección de la afirmación no es inmediatamente obvia. También es bastante quisquilloso señalar que su corrección solo se prueba en la geometría euclidiana. Pero siempre pensé que era al menos fácil de ver, ya que obviamente los ángulos exteriores de cualquier n-ágono convexo suman 360+180n grados. Así que me quedé desconcertado cuando me enteré de las geometrías no euclidianas. Ahora no estoy seguro de qué hacer con esta respuesta. ¿Debo editarlo/eliminarlo o simplemente dejarlo como está?
Bueno, si dejamos que la palabra triángulo sea un polígono de 2 dimensiones y 3 lados. Entonces sí, porque si aumentamos las longitudes de un segmento de línea, también debe aumentar, piense si esto no fuera cierto, significaría que un triángulo podría tener dos ángulos obtusos porque con la propiedad de todos los ángulos internos suman 180 grados. requeriría si el ángulo fuera de 91 grados que los otros sumen 89 grados, lo cual no es obtuso. Veo que apuntas a un tipo especial de espacio, pero debido a que en el espacio en el que vivimos funciona de la manera en que lo hace, los ángulos internos de un triángulo suman 180 grados... (Voy a continuar más abajo).
Continuando... Esto también porque un triángulo es propiedad de nuestro espacio, según su definición debido a la expansión y reducción de las longitudes de los triángulos necesarias para cambiar los ángulos del triángulo, lo que haría que no fuera un polígono porque tendría que ser una forma abierta.

Si un cálculo proposicional A contiene todos los teoremas del cálculo proposicional B bajo separación y sustitución uniforme de variables proposicionales, pero B no contiene todos los teoremas de A, entonces uno de los axiomas simples más cortos de A es más largo que cualquiera de los axiomas simples más cortos. axiomas de B. O uno podría decir más al azar "si el cálculo proposicional A es más grande que el cálculo proposicional B, entonces uno de los axiomas individuales más cortos de A es más largo que cualquiera de los axiomas individuales más cortos de B".

¿Ha establecido el "teorema" falso, o es una verdadera declaración contraria a la intuición? (Claramente, en realidad no estoy intuyendo nada :-))
@MarkHurd Es un "teorema falso". El cálculo implicacional puro tiene un axioma único de 13 letras, pero los axiomas únicos para, por ejemplo, BCI son más largos.
Buena suerte para encontrar un profano cuyos ojos no se pongan vidriosos ante el término "cálculo proposicional", y mucho menos uno que encuentre algo aquí obvio.
@rumtscho No trabajo en la academia. También solo tomé un curso de lógica en la universidad y 2 cursos de cálculo.

Tal vez esto no sea exactamente lo que estabas buscando, ¡pero sigue siendo divertido! ¿Qué tal una prueba que obviamente es incorrecta pero (para los recién llegados) es difícil averiguar qué es lo que está mal?

Dejar X = y . Entonces

X 2 = X y
X 2 y 2 = X y y 2
( X + y ) ( X y ) = y ( X y )
X + y = y
2 y = y
2 = 1

sí, puedes "probar" todo tipo de cosas cuando divides por 0... ;)

Un teorema "obviamente verdadero":

Si toma un objeto 3d tu y cortarlo en un número finito de piezas, cualquier objeto V Reorganizo esas piezas para que tengan el mismo volumen que el objeto. tu comencé con

Pero, de hecho, la paradoja de Banach-Tarski nos dice que esto no es cierto, si construimos nuestros subconjuntos de número finito de tu bastante "extrañamente", en realidad podemos construir un V con cualquier volumen que nos gustaría.

Cualquier función continua es diferenciable al menos en alguna parte, ¿verdad?

Falso, la función de Wierstrass es un famoso contraejemplo

https://en.wikipedia.org/wiki/Weierstrass_function

Es continuo en todas partes, pero no es diferenciable en ninguna parte.

Una función analítica con soporte compacto se desvanece de forma idéntica.

Eso es realmente cierto, mientras permanezcamos en C norte resp. R norte . Se vuelve falso cuando uno toma una variedad analítica compacta, pero luego es obvio que es falso.

Lebesgue dijo una vez que las proyecciones de Borel se establecen en R 2 sobre uno de sus ejes se encuentran también decorados de Borel. Este hecho es en realidad falso, cuya realización se atribuye al matemático de corta vida Mikhail Yakovlevich Suslin.

Desafortunadamente es muy difícil encontrar un contraejemplo. El único que he visto tiene un resultado en la teoría descriptiva de conjuntos sobre norte norte y utiliza el hecho de que es homeomorfo para R .

Espero que estés recordando mal o malinterpretando los argumentos que has visto, porque norte norte no es homeomorfo a R . Por ejemplo, uno está conectado y el otro no. Uno es σ -compacto y el otro no lo es.
Eso depende de la topología, pero podría ser que el argumento en el que estoy pensando usó el espacio de todas las secuencias (tanto finitas como infinitas) de números naturales o desarrolló el homeomorfismo con la exclusión de q .
"Eso depende de la topología". Obviamente. "El espacio de todas las sucesiones (tanto finitas como infinitas) de números naturales". Tal vez, pero eso no es norte norte . "Con exclusión de q ." Pero entonces eso no es R . De todos modos, una o dos palabras aclarando la imprecisión no estarían de más.
@jjfunk: ¿Es posible que el mapa del que hablas sea el homeomorfismo entre el espacio de Baire y los irracionales dados por fracciones continuas infinitas?

El sueño del estudiante de primer año:

( X + y ) pag = X pag + y pag
.

Obviamente falso. Pero cierto en característica pag .